Download as pdf or txt
Download as pdf or txt
You are on page 1of 194

Question 1

2 / 2 pts
Which of the following statements is/are correct?
I. In the audit risk equation, detection risk is the controllable variable.
II. The two factors that contribute to the probability that unaudited financial statements
contain material errors or fraud are inherent risk and control risk.
Correct!

Both I & II

Neither I nor II

II only

II only

Question 2
2 / 2 pts
The auditor communicates the results of his or her work through the medium of the

Engagement letter.

Management letter.

Correct!

Audit report.

Financial statements.
Question 3
2 / 2 pts
A CPA establishes QC policies and procedures for deciding whether to accept a new client
or continue to perform services for a current client. The primary purpose for establishing
such policies is to

Comply with the quality control standards established by regulatory bodies.

Ensure that the client is capable of financing the resources needed in the audit
engagement.

Enable the auditor to attest to the integrity or reliability of a client.

Correct!

Minimize the likelihood of association with clients whose management lacks integrity

Question 4
2 / 2 pts
Which of the following conditions does not increase inherent risk?

The client has entered numerous related party transactions during the year under audit.

The board of directors approved a substantial bonus for the president and chief executive
officer and also approved an attractive stock option plan for themselves.

Correct!

The client hired an employee who was a former employee of its major competitor.
The client has lost a major customer accounting for approximately 40% of annual
revenues.

Question 5
0 / 2 pts
Which of the following best describes the reason why an independent auditor reports on
financial statements?

Poorly designed internal control may exist.

Correct Answer

A management fraud may exist and is more likely to be detected by independent auditors.

A misstatement of account balances may exist and is generally corrected as the result of
the independent auditor's work.

You Answered

Different interests may exist between the company preparing the statements and the
persons using the statements.

Question 6
2 / 2 pts
Which of the following is the correct order of steps in the audit process?
I. Prepare engagement letter
II. Perform tests of control
III. Develop an overall strategy for the expected conduct and scope of audit
IV. Completing the audit
V. Perform substantive tests
Correct!

I, III, II, V, IV

I, II, III, V, IV

I, III, IV, II, V

I, III, V, II, IV

Question 7
2 / 2 pts
On the basis of the audit evidence gathered and evaluated, an auditor decides to increase
the assessed level of control risk from that originally planned. To achieve an overall audit
risk level that is substantially the same as the planned audit risk level, the auditor would

Increase inherent risk.

Decrease substantive testing.

Correct!

Decrease detection risk.

Increase materiality levels.

Question 8
2 / 2 pts
The acceptable level of detection risk and the combined level of inherent risk and control
risk are _________ related.

not

directly

Correct!

inversely

proportionately

Question 9
0 / 2 pts
The overall objectives of the auditor in conducting an audit of financial statements are
I. To obtain reasonable assurance about whether the financial statements as a whole are
free from material misstatements, whether caused by fraud or error
II. To report on the financial statements
III. To obtain persuasive rather than conclusive evidence
IV. To detect all misstatements, whether due to fraud or error

I and II only

You Answered

None of the other choices

I, II, III and IV


Correct Answer

I, II and III only

II and IV only

Question 10
2 / 2 pts
Which of the following has the primary responsibility for the fairness of the
representations made in the financial statements?

Audit committee

Correct!

Client's management

PRC-Board of Accountancy

Independent auditor

Question 11
0 / 2 pts
Assuming you an auditor who was approached by a new client to perform an audit for the
first time, you should make inquiries of the predecessor auditor. This is a necessary
procedure because the predecessor may be able to provide you with information that will
assist you in determining
You Answered

Whether, in the predecessor's opinion, internal control of the company has been
satisfactory.
Whether the predecessor's work should be utilized.

Correct Answer

Whether the engagement should be accepted.

Whether the company follows the policy of rotating its auditors.

Question 12
2 / 2 pts
Warning signs that cause the auditor to question management integrity must be taken
seriously and pursued vigorously. Which of the following may lead the auditor to suspect
management dishonesty?
Correct!

The president and chief executive officer of the client corporation has held numerous
meetings with the controller for the purpose of discussing accounting practices that will
maximize reported profits.

A new regulation making customer licenses more difficult to obtain may adversely affect
the client's operations.

The client has been named as a defendant in a product liability suit.

The client has experienced a decrease in revenue from increased import competition.

Question 13
2 / 2 pts
The state of mind that permits the expression of a conclusion without being affected by
influences that compromise professional judgment.
Professional Skepticism

Objectivity

Independence in Appearance

Correct!

Independence of Mind

Question 14
2 / 2 pts
Which of the following is not a component of audit planning?

Obtaining an understanding of the business.

Developing audit programs.

Correct!

Observing the client's annual physical inventory taking and making test counts of selected
items.

Making arrangements with the client concerning the timing of audit field work and use of
the client's staff in completing certain phases of the examination.

Question 15
2 / 2 pts
Users of financial statements demand independent audit because
Users expect auditors to correct management errors.

Correct!

Management may not be objective in reporting.

Users demand assurance that fraud does not exist.

Management relies on the auditor to detect error and fraud.

Question 16
2 / 2 pts
An audit involves ascertaining the degree of correspondence between assertions and
established criteria. In the case of an audit, of financial statements, which of the following
would not be considered as suitable criteria?

Philippine Financial Reporting Standards

Correct!

Philippine Standards on Auditing

International Accounting Standards

Philippine Accounting Standards

Question 17
2 / 2 pts
Which of the following standards deals with agreeing the terms of audit?
PSA 300

PSA 220

PSA 200

Correct!

PSA 210

Question 18
2 / 2 pts
Which of the following conditions supports an increase in detection risk?

Internal control over shipping, billing, and recording of sales revenue is weak.

Correct!

Internal control over cash receipts is excellent.

Application of analytical procedures reveals a significant increase in sales revenue in


December, the last month of the calendar year.

Study of the business reveals that the client recently acquired a new company in an
unrelated industry.

Question 19
2 / 2 pts
You are the auditor assigned to AB Corp. for the year-ended December 31, 2020. Which
of the following statements best describes your responsibility regarding the detection of
fraud as the auditor?

An auditor is responsible for the failure to detect fraud only when an unqualified opinion
is issued.

An auditor is responsible for the failure to detect fraud only when such failure clearly
results from nonperformance of audit procedures specifically described in the
engagement letter.

Correct!

An auditor should design audit procedures that will provide reasonable assurance that the
financial statements are free from material misstatement due to errors and/or fraud.

An auditor must extend auditing procedures to actively search for evidence of fraud
where the examination indicates that fraud may exist.

Question 20
2 / 2 pts
Which of the following matters would an auditor most likely consider when establishing
the scope of the audit?

Audit areas where the is high risk of material misstatement

Correct!

The expected audit coverage, including the number and locations of the entity’s
components to be included.

The entity’s time table for reporting


The entity’s time table for reporting

Question 21
2 / 2 pts
The understanding and agreement between the client and the auditor as to the degree of
responsibilities to be assumed by each are normally set forth in a(an)

audit report.

representation letter.

Correct!

engagement letter.

management letter.

Question 22
2 / 2 pts
When an auditor believes that an understanding with the client has not been established,
he or she should ordinarily

Assess control risk at the maximum level and perform a primarily substantive audit.

Perform the audit with increase professional skepticism.

Correct!

Decline to accept or perform the audit.


Modify the scope of the audit to reflect an increased risk of material misstatement due to
fraud.

Question 23
2 / 2 pts
What is the responsibility of a successor auditor to communicate with the predecessor
auditor in connection with a prospective new client?

The successor auditor need not contact the predecessor if the successor is aware of all
available relevant facts.

The successor auditor has no responsibility to contact the predecessor auditor.

Correct!

The successor auditor should obtain permission from the prospective client to contact the
predecessor auditor.

The successor auditor should contact the predecessor auditor if the client authorizes
contact.

Question 24
2 / 2 pts
In pursuing its quality control objectives with respect to acceptance of a client, a CPA firm
is not likely to

Review financial statements of the proposed client.

Make inquiries of the proposed client's legal counsel.


Correct!

Review the personnel practices of the proposed client.

Make inquiries of previous auditors.

Question 25
2 / 2 pts
Which of the following statements is/are correct?
I. According to PSA 300, the auditor may still change the overall audit strategy and the
audit plan during the course of the audit.
II. According to PSA 300, the auditor may organize meetings with management and those
charged with governance to discuss the nature, timing and extent of the audit work.

Neither I nor II

Correct!

Both I & II

II only

I only

Question 26
2 / 2 pts
If it is probable that the judgment of a reasonable person would have been changed or
influenced by the omission or misstatement of information, then the information is

Significant.
Correct!

Material.

Relevant.

Pervasive.

Question 27
2 / 2 pts
Which of the following statements is/are correct?
I. An auditor may not eliminate audit risk because of the inherent limitations of audit but it
may be reduced by doing more audit procedures.
II. Engagement risk may be eliminated by not accepting a particular client.

Neither I nor II

II only

I only

Correct!

Both I & II

Question 28
2 / 2 pts
Which of the following statements is/are correct?
I. During the risk assessment phase, the auditor shall design the overall responses to the
assessed risk of material misstatement.
II. In risk response phase, the auditor shall evaluate the audit evidence obtained to
determine if additional audit procedures are required to be done.

II only

Correct!

Neither I nor II

Both I & II

I only

Question 29
0 / 2 pts
Which of the following statements is/are correct?
I. Reasonable assurance means that the auditor guarantees that the financial statements
are free from material misstatements.
II. Risk-based audit approach is different from account-based approach because it begins
with the assessment of the types and likelihood of misstatements in the financial
statements as a whole.

Both I & II

Correct Answer

II only

You Answered

Neither I nor II
I only

Question 30
2 / 2 pts
Which of the following statements is/are correct?
I. Financial risk and business risk are risks that exist before the audit engagement.
II. Business risk may be affected by the geographic location of the entity, the economic
climate and technological changes.
Correct!

Both I & II

II only

Neither I nor II

I only

Question 1
0 / 1 pts
The overall objectives of the auditor in conducting an audit of financial statements are:
I. To obtain reasonable assurance about whether the financial statements as a whole are
free from material misstatement, whether caused by fraud or error.
II. To report on the financial statements.
III. To obtain conclusive rather than persuasive evidence
IV. To detect all misstatements, whether due to fraud or error
Correct Answer

I and II only

You Answered

I, II, and III only

I, II, III and IV

None of the choices.

II and IV only

Question 2
1 / 1 pts
Which of the following statements best describes the auditor's responsibility regarding
the detection of material errors and frauds?

Extended auditing procedures are required to detect unrecorded transactions even if


there is no evidence that material errors and frauds may exist

The auditor is responsible for the failure to detect material errors and frauds only when
such failure results from the misapplication of generally accepted accounting principles.

Correct!

The audit should be designed to provide reasonable assurance that material errors and
frauds are detected.
The auditor is responsible for the failure to detect material errors and frauds only when
the auditor fails to confirm receivables or observe inventories.

Question 3
0 / 1 pts
You were assigned as the auditor of Papasa Company. After studying and evaluating its
existing internal control, you concluded that the policies and procedures are designed and
functioning as intended. As the auditor, you would most likely
You Answered

set the desired audit risk in a lower level than would be set under conditions of weak
internal control.

Correct Answer

set detection risk at a higher level than would be set under conditions of weak internal
control.

determine the control policies and procedures that should prevent or prevent fraud and
error.

perform further control tests to check if the controls can be relied upon for the purpose of
audit.

Question 4
1 / 1 pts
The audit risk model is used primarily

to evaluate the evidence which has been gathered.

while doing tests of controls.


to determine the type of opinion to express.

Correct!

for planning purposes in determining how much evidence to accumulate.

Question 5
0 / 1 pts
Which of the following statements is/are correct?
I. Detection risk is a function of the efficiency of an audit procedure.
II. The existing levels of inherent risk, control risk and detection risk can be changed at the
discretion of the auditor.

I only

Correct Answer

Neither I nor II

You Answered

II only

Both I & II

Question 6
1 / 1 pts
On the basis of audit evidence gathered and evaluated, an auditor decides to increase the
assessed level of control risk, and therefore the risk of material misstatement, from that
originally planned. To achieve an overall audit risk level that is substantially the same as
the planned audit risk level, the auditor would:
Decrease substantive testing

Increase inherent risk

Correct!

Decrease detection risk

Increase materiality levels

Question 7
1 / 1 pts
The acceptable financial reporting framework in the Philippines are those issued and
promulgated by the:
Correct!

FRSC

AASC

PICPA

COA

Question 8
1 / 1 pts
Which of the following statements is/are correct?
I. Audit risk is the risk that the auditor gives an appropriate audit opinion when the
financial statements are materially misstated.
II. Detection risk is the risk that an auditor’s substantive procedures will detect a
misstatement that exists in an account balance or class of transactions that could be
material, individually or when aggregated with misstatements in other balances or classes.

Both I & II

II only

I only

Correct!

Neither I nor II

Question 9
0 / 1 pts
Inherent risk and control risk differ from detection risk in that inherent risk and control
risk are:
You Answered

Changes at the auditor’s discretion while detection risk is not

Correct Answer

Functions of the client and its environment while detection risk is not

Considered at the individual account balance level while detection risk is not.

Elements of audit risk while detection risk is not


Question 10
1 / 1 pts
The following are examples of conditions and events that may indicate the existence of
risks of material misstatement, except

Application of new accounting pronouncements.

Pending litigation and contingent liabilities.

Correct!

Operations in regions that are economically stable

Entities or business segments likely to be sold.

Question 11
1 / 1 pts
What is the estimated level of audit risk if inherent risk is .50, control risk .40, and
detection risk .10?

None of the choices.

.10

Correct!

.02

.20
.04

Question 12
1 / 1 pts
Which of the following statements is/are correct?
I. Detection risk cannot be changed at the auditor’s discretion.
II. The greater the inherent and control risks the auditor believes exists, the less detection
risk that can be accepted.

Neither I nor II

Both I & II

I only

Correct!

II only

Question 13
1 / 1 pts
Which of the following statements is/are correct?
Statement I: If there is a higher level of assessed risk of material misstatement, an auditor
should also set the detection risk at a higher level.
Statement II: If there is a lower level of assessed risk of material misstatement, an auditor
should also set the detection risk at a lower level.
Correct!

Neither I nor II
II only

Both I & II

I only

Question 14
1 / 1 pts
As the acceptable level of detection risk decreases, an auditor may

change the timing of substantive tests by performing them at an interim date rather than
at year-end.

Correct!

change the nature of substantive tests from a less effective to a more effective procedure.

change the timing of tests of controls by performing them at several dates rather than at
one time.

reduce substantive testing by relying on the assessments of inherent risk and control risk.

Question 15
0 / 1 pts
The risk that the audit will fail to uncover a material misstatement is eliminated

if the client is implementing effective internal control to detect fraud and errors.

You Answered
if the company’s internal audit is able to attain its purpose as an independent party inside
the company that monitors and assesses factors such as the internal control, safeguarding
of assets, financial reporting and compliance.

when the auditor complied with PSAs and the client complied with PFRS.

Correct Answer

is not possible in any circumstances.

Question 16
0 / 1 pts
Which of the following statements is/are correct?
I. Detection risk is a function of the effectiveness of an auditing procedure and its
application.
II. Detection risk exists independently of the audit of the financial statements.

Neither I nor II

Correct Answer

I only

You Answered

II only

Both I & II

Question 17
0 / 1 pts
Regarding financial instruments, which of the following industries has the higher level of
risk of material statement?
You Answered

Mining

Manufacturing

Correct Answer

Banking

Airline

Question 18
1 / 1 pts
Generally, the most difficult type of misstatement to detect is fraud based on
Correct!

non recording of business transactions.

recording of transactions which never occurred.

numerous related party transactions wherein incomplete supporting documents are


available.

misappropriation of assets accomplished by a group of employees.

Question 19
1 / 1 pts
An auditor uses the assessed level of control risk to
Correct!

determine the acceptable level of detection risk for financial statements.

evaluate the effectiveness of the entity’s internal control policies and procedures.

indicate whether materiality threshold for planning and evaluation purposes are
sufficiently high.

identify transactions and account balances where inherent risk is at a high level.

Question 20
1 / 1 pts
Which of the following statements is/are correct?
I. Cash is more susceptible to theft than an inventory of coal because it has a greater
inherent risk.
II. The risk that material misstatement will not prevented or detected on a timely basis by
internal control can be reduced to a zero by effective controls.

Neither I nor II

II only

Correct!

I only
Both I & II

Question 21
0 / 1 pts
Which of the following is/are example/s of error?
I. Transactions without substance were deliberately recorded.
II. Some small value assets were misappropriated.
III. The effects of the transactions have been omitted from the records.
IV. Incorrect accounting estimates due to misinterpretation of facts.

II and IV only

Correct Answer

IV only

You Answered

II, III and IV only

None of the choices.

I, II and IV only

Question 22
1 / 1 pts
Philippine Standards on Auditing should be looked upon by practitioners as

ideals to strive for, but which are not achievable.


maximum standards to which the conduct of audit be compared to.

Correct!

minimum standards of performance which must be achieved in an audit.

benchmark to be used on all audits and reviews done by auditors

Question 23
1 / 1 pts
The auditor’s responsibility is to conduct the audit according to:

International Accounting Standards

Philippine Financial Reporting Standards

Correct!

Philippine Standards on Auditing

Accepted Accounting Standards

Question 24
1 / 1 pts
Inherent risk and control risk differ from detection risk in that they
Correct!

Exist independently of the financial statement audit.


Can be changed at the auditor’s discretion.

May be assessed in either quantitative or nonquantitative terms.

Arise from the misapplication of auditing procedures.

Question 25
1 / 1 pts
Which statement is correct regarding business risks?
I. The risk of material misstatement of the financial statements is broader than business
risk, though it includes the latter.
II. A business risk may have an immediate consequence for the risk of misstatement for
classes of transactions, account balances, and disclosures at the assertion level or the
financial statements as a whole.

I only

Neither I nor II

Both I & II

Correct!

II only

Question 26
1 / 1 pts
Why would the auditor assess control risk?
Because it determines whether sampling risk is sufficiently low

Because it includes the aspects of nonsampling risk that are controllable.

Correct!

Because it affects the level of detection risk the auditor may accept.

Because it indicates where inherent risk may be the greatest

Question 27
1 / 1 pts
Which of the following is an example of fraudulent financial reporting?
Correct!

Company management changes inventory count tags and overstates ending inventory,
while understating cost of goods sold.

The treasurer diverts customer payments to his personal due, concealing his actions by
debiting an expense account, thus overstating expenses.

An employee steals inventory and the “shrinkage” is recorded in cost of goods sold

An employee steals small tools from the company and neglects to return them; the cost is
reported as a miscellaneous operating expense.

Question 28
1 / 1 pts
With regards to making accounting estimates, which of the following industries poses the
highest risk of material misstatement?
Correct!

Mining

Airline

Banking

Manufacturing

Question 29
0 / 1 pts
Which of the following statements is/are correct?
I. Engagement team includes all partners, staff, any individuals engaged by the firm who
perform procedures on the engagement including external experts.
II. “Firm” means a sole practitioner, partnership or corporation of accountants.

Both I & II

You Answered

II only

Correct Answer

Neither I nor II

I only
Question 30
1 / 1 pts
When an independent auditor is approached to perform an audit for the first time, he or
she should make inquiries of the predecessor auditor. Inquiries are necessary because
the predecessor may be able to provide the successor with information that will assist the
successor in determining whether

The company rotates auditors.

The predecessor’s work should be used.

In the predecessor’s opinion, control risk is low.

Correct!

The engagement should be accepted.

Question 31
0 / 1 pts
For recurring audit, the auditor shall perform the following activities.
I. Performing procedures required by PSA 220, “Quality Control for Audits of Historical
Financial Information” regarding the continuance of the client relationship and the
specific audit engagement.
II. Evaluating compliance with ethical requirements, including independence, as required
by PSA 220.
III. Establishing an understanding of the terms of the engagement, as required by PSA 210,
“Terms of Audit Engagements”.
IV. Performing procedures required by PSA 220 regarding the acceptance of the client
relationship and the specific audit engagement.
V. Communicating with the predecessor auditor, where there has been a change of
auditors, in compliance with relevant ethical requirements.
I,II,III,IV,V

I,III,V

You Answered

I,II,III,IV

None of the choices.

Correct Answer

I,II,III

Question 32
1 / 1 pts
Which of the following statements is/are correct?
I. As part of preliminary engagement activities, the engagement partner shall form a
conclusion on compliance with independence requirements that apply to the audit
engagement.
II. The preconditions for an audit shall be present for an engagement to be accepted.

I only

II only

Correct!

Both I & II
Neither I nor II

Question 33
1 / 1 pts
According to PSA 210, Which of the following statements is/are correct?
I. The engagement letter assists in the supervision and review of the audit work.
II. The auditor may agree to a change of engagement where there is reasonable
justification for doing so.
Correct!

II only

Neither I nor II

I only

Both I & II

Question 34
0 / 1 pts
For initial audit engagements, the auditor shall perform the following activities.
I. Performing procedures required by PSA 220, “Quality Control for Audits of Historical
Financial Information” regarding the continuance of the client relationship and the
specific audit engagement.
II. Evaluating compliance with ethical requirements, including independence, as required
by PSA 220.
III. Establishing an understanding of the terms of the engagement, as required by PSA 210,
“Terms of Audit Engagements”.
IV. Performing procedures required by PSA 220 regarding the acceptance of the client
relationship and the specific audit engagement.
V. Communicating with the predecessor auditor, where there has been a change of
auditors, in compliance with relevant ethical requirements.

None of the choices.

I,II,III

You Answered

I,II,III,IV

I,III,V

Correct Answer

I,II,III,IV,V

Question 35
1 / 1 pts
Which of the following least likely requires the auditor to send a new engagement letter?

Any revised or special terms of the engagement.

Legal requirements and other government agencies’ pronouncements.

Correct!

A recent change in the audit firm’s management.


An indication that the client misunderstands the objective and scope of the audit.

Question 36
1 / 1 pts
If permission from client to discuss its affairs with the proposed auditor is denied by the
client, the predecessor auditor should:
Correct!

Disclose the fact that the permission to disclose is denied by the client.

Keep silent of the denial.

Disclose adequately to proposed auditor all noncompliance made by the client.

Seek legal advice before responding to the proposed auditor.

Question 37
1 / 1 pts
Prior to the acceptance of an audit engagement with a client who has terminated the
services of the predecessor auditor, the CPA should

Accept the engagement without contacting the predecessor auditor since the CPA can
include audit procedures to verify the reason given by the client for the termination.

Correct!

Advise the client of the intention to contact the predecessor auditor and request
permission for the contact
Not communicate with the predecessor auditor because this would in effect be asking the
auditor to violate the confidential relationship between auditor and client.

Contact the predecessor auditor without advising the prospective client and request a
complete report of the circumstance leading to the termination with the understanding
that all information disclosed will be kept confidential.

Question 38
1 / 1 pts
If the preconditions for an audit were not satisfied, the auditor shall communicate the
matter with:
Correct!

the management

None of the choices

the internal audit department head

the authorities

Question 39
1 / 1 pts
According to PSA 210 Agreeing the Terms of Audit Engagement, audit engagement letter
shall not include:

The scope of audit

The objective of audit


Correct!

The responsibilities of the government

The responsibilities of the auditor

Question 40
1 / 1 pts
According to PSA 210, Which of the following statements is/are correct?
I. The auditor and the client need not agree on the terms of the engagement as long as the
auditor prepares the engagement letter.
II. Where the terms of the engagement are changed, the auditor and the client need not
agree on the new terms if they already agreed on the old terms

Both I & II

I only

II only

Correct!

Neither I nor II

Question 41
1 / 1 pts
Which of the following least likely influence the auditor’s decision to send a separate
engagement letter to a component of parent entity client?
Legal requirements.

Correct!

Location of the principal place of business of the component entity.

Degree of ownership over a component entity by parent company.

Who appoints the auditor of the component.

Question 42
1 / 1 pts
Ariel, CPA was engaged to audit the financial statements of Toffee Company, a company
publicly listed on Philippine Stock Exchange. In relation to preconditions for an audit,
which of the following is an acceptable financial reporting framework to be applied in the
preparation of the financial statements?
Correct!

Philippine Financial Reporting Standards.

Philippine Standards on Auditing.

Philippine Institute of CPA Standards

Generally Accepted Auditing Standards.

Question 43
1 / 1 pts
Audit Engagement Letter are prepared by:
The chairman of the board

None of the above. Engagement letter are pre-formatted letters where both parties
(management and auditor) just fill up the form.

The head or if not applicable any representative of the audit committee

Correct!

The audit firm engaged for the audit

Question 44
1 / 1 pts
Which of the following is least likely included in an audit engagement letter?

Management responsibility for the financial statements.

The form of any reports or other communication of the results of the engagement.

Correct!

The objective of financial reporting.

Arrangement concerning the involvement of other auditors or experts in some aspects of


the audit.

Question 45
1 / 1 pts
Which of the following is not part of the responsibilities of the auditor as outlined in the
terms of the audit engagement?

To conduct the audit engagement in accordance with Philippine Standards on Auditing.

The identification and assessment of material misstatement of the financial statements.

To evaluate the appropriateness of accounting policies.

Correct!

The preparation and presentation of the financial statements.

Question 46
1 / 1 pts
When a change in the type of engagement from higher to lower level of assurance is
reasonably justified, the report based on the revised engagement

Should contain a separate paragraph that refers to the original engagement.

Should qualify the opinion due to scope limitation.

Should always refer to any procedures that may have been performed in the original
engagement.

Correct!

Omits reference to the original engagement

Question 47
1 / 1 pts
Which of the following is not a valid reason for a change of the engagement to a lower
“level of assurance”?
Correct!

Restriction on the scope of the engagement.

The client’s need is satisfied by an engagement that provides lower level of assurance.

Change in circumstances affecting the need for the service.

Misunderstanding as to the nature of the engagement originally requested.

Question 48
1 / 1 pts
After accepting an engagement, a practitioner is not allowed to change the engagement to
a non-assurance engagement, except when there is a reasonable justification for the
change. Which of the following ordinarily will justify a request for such change?
I. A change in circumstances that affects the intended user’s requirements.
II. A misunderstanding concerning the nature of the engagement.

Neither I nor II

I only

Correct!

Both I & II
II only

Question 49
1 / 1 pts
The use of an engagement letter is the best method of documenting
I. notification of any changes in the original arrangements of the audit.
II. the description of any letters or reports that the auditor expects to issue.
III. the required communication of significant deficiencies in internal control structure.
IV. significantly lower materiality levels than those used in the prior audit.

II and IV only.

None of the choices.

I, II and III only.

I, II and IV only.

Correct!

I and II only.

Question 50
1 / 1 pts
Which of the following is correct statement?

The subsequent discovery that a material misstatement exists in the financial statements
is evidence of inadequate planning, performance, or judgment on the part of the auditor.
The auditor should use professional judgment to assess audit risk and to design audit
procedures to ensure it is eliminated.

None of the statements is correct.

The auditor is an insurer, and his or her report constitutes a guarantee.

Correct!

The auditor should obtain an understanding of the accounting and internal control
systems sufficient to plan the audit and develop an effective audit approach

Question 51
1 / 1 pts
Which of the following should an auditor obtain from the predecessor
auditor prior to accepting an audit engagement?

All of the given choices.

Correct!

Facts that might bear on the integrity of management.

All matters of continuing accounting significance.

Analysis of balance sheet accounts.

Analysis of income statement accounts.


Question 52
0 / 1 pts
The following are valid reasons why an auditor sends to his client an engagement letter:

Avoid
Confirms the
misunderstanding Assures CPA’s
auditor’s acceptance
with respect to
of the appointment Objective and scope compliance to
engagement GAAS
of the audit

a. YES YES YES YES

b. YES YES YES NO

c. NO YES YES NO

d. YES NO YES YES

You Answered

Correct Answer

Question 53
1 / 1 pts
A CPA establishes quality control policies and procedures for deciding whether to accept
a new client or continue to perform services for a current client. The primary purpose for
establishing such policies and procedures is

To enable the auditor to attest to the integrity or reliability of a client.

To lessen the exposure to litigation resulting from failure to detect irregularities in client
financial statements.

To comply with the quality control standards established by regulatory bodies.

Correct!

To minimize the likelihood of association with clients whose management lacks integrity.

Question 54
1 / 1 pts
Faith Company engaged King, Rico, and Scarlet Company to audit the company’s financial
statement. Accordingly, which of the following is the responsibility of the management of
Faith Company?

Communicate in writing concerning any significant deficiencies in internal control


relevant to the audit of the financial statements that were identified during the audit.

To identify and assess the risks of material misstatement of the financial statements of
Faith company, whether due to fraud or error, design and perform audit procedures
responsive to those risks.

Correct!

To provide King, Rico and Scarlet Company access to all information of which
management is aware that is relevant to the preparation of the financial statements such
as records, and documentation.
Obtain an understanding of internal control of Faith Company relevant to the audit to
design audit procedures that are appropriate in the circumstances, but not for expressing
an opinion on the effectiveness of the entity’s internal control.

Question 55
0 / 1 pts
In planning an audit engagement, which of the following affects the independent auditor's
judgment as to the quantity, type, and content of working papers?
Correct Answer

The anticipated nature of the auditor's report

You Answered

The estimated occurrence rate of attributes.

The content of the client's representation letter.

The preliminary evaluations based on substantive testing.

Question 56
1 / 1 pts
Client strategy templates provide a means for an auditor to
Correct!

Compile the strategies and characteristics of an entity.

Acquire, evaluate, and document evidence.


Distinguish between engagement risk and audit risk.

Focus on transactions likely to affect audit risk.

Question 57
1 / 1 pts
It involves establishing the overall audit strategy for the engagement and developing an
audit plan in order to reduce audit risk to an acceptably low level.

Organizing

Directing

Reporting

Correct!

Planning

Question 58
1 / 1 pts
Which of the following matters would an auditor most likely consider when establishing
the scope of the audit?

The discussion with the entity’s management concerning the expected communications
on the status of audit work throughout the engagement and the expected deliverables
resulting from the audit procedures.

The entity’s timetable for reporting, such as at interim and final stages.
Correct!

The expected audit coverage, including the number and locations of the entity’s
components to be included.

Audit areas where there is a higher risk of material misstatement.

Question 59
1 / 1 pts
Which of the following statements is/are correct?
I. According to PSA 300, the auditor may no longer change the overall audit strategy and
the audit plan during the course of the audit.
II. According to PSA 300, the auditor may organize meetings with management and those
charged with governance to discuss the nature, timing and extent of the audit work.
III. As defined in PSA 315 “Significant risk” means an identified and assessed risk of
material misstatement that, in the auditor’s judgment, requires special audit
consideration.

I and II only

Correct!

II & III only

I and III only

All of the statements,

Question 60
1 / 1 pts
Which of the following procedures would an auditor least likely perform in planning a
financial statement audit?

Discussing matters that may affect the audit with firm personnel responsible for non-
audit services to the entity.

Correct!

Selecting a sample of vendor’s invoices for comparison to receiving reports.

Reading the current year’s interim financial statements.

Coordinating the assistance of entity personnel in data preparation.

Question 61
1 / 1 pts
Which of the following matters should be considered by the auditor in developing the
overall audit strategy?
I. Important characteristics of the entity, its business, its financial performance and its
reporting requirements including changes since the date of the prior audit.
II. Conditions requiring special attention, such as the existence of the related parties.
III. The setting of materiality level for audit purposes.
Correct!

I, II & III

I & III

I & II
I only

None of the choices.

Question 62
1 / 1 pts
The existence of a related party transaction may be indicated when another entity

Sells real estate to the corporation at a price that is comparable to its appraised value.

Correct!

Absorbs expenses of the corporation.

Borrows from the corporation at a rate of interest which equals the current market rate.

Lends to the corporation at a rate of interest, which equals the current market rate.

Question 63
1 / 1 pts
The extent of planning will vary according to any of the following, except:

Size of the audit client.

Correct!

The assessed level of control risk.


Auditor's experience with the entity and knowledge of the business.

The nature and complexity of the audit engagement.

Question 64
1 / 1 pts
Which of the following situations would most likely require special audit planning by the
auditor?

Some items of factory and office equipment do not bear identification numbers.

Correct!

Inventory comprises precious stones.

Assets costing less than P500 are expensed even though the expected life exceeds one
year.

Depreciation methods used on the client's tax return differ from those used on the books.

Question 65
1 / 1 pts
Early appointment of the auditor enables preliminary work to be performed by the
auditor, which
benefits the client in that it permits the examination to be performed in

A more thorough manner.

Accordance with quality control standards.


Accordance with generally accepted auditing standards.

Correct!

A more efficient manner.

Question 66
1 / 1 pts
An independent auditor observed that only one of the company's ten divisions had a large
number of material sales transactions close to the end of the fiscal year. In terms of risk
analysis, this would most likely lead the auditor to conclude that:

Risks associated with auditing this division are not affected by this information.

Correct!

There is a relatively higher risk of overstatement of revenues for this division than for
other divisions.

There is a high risk that liabilities of this division are understated.

There is a high risk that the other nine divisions have understated revenues.

Question 67
0 / 1 pts
The auditor shall develop an audit plan that shall include a description of:
I. The nature, timing and extent of planned risk assessment procedures
II. The nature, timing and extent of planned further audit procedures at the assertion level
III. Other planned audit procedures that are required to be carried out so that the
engagement complies with PSAs
II, III

You Answered

Correct Answer

I,II,III

None of the choices

I, II

Question 68
0 / 1 pts
The auditor shall document:
I. The overall audit strategy
II. The audit plan
III. Any significant changes made during the audit engagement to the overall audit
strategy or the audit plan, and the reasons for such changes
IV. The approval of the client in the changes made.

I, II ,III

None of the choices

Correct Answer

I
You Answered

I, II, III, IV

I, II

Question 69
1 / 1 pts
In establishing the overall audit strategy, the auditor shall:
I. Identify the characteristics of the engagement that define its scope
II. Ascertain the reporting objectives of the engagement to plan the timing of the audit and
the nature of the communications required
III. Consider the factors that, in the auditor’s professional judgment, are significant in
directing the engagement team’s efforts
IV. Consider the results of preliminary engagement activities and, where applicable,
whether knowledge gained on other engagements performed by the engagement partner
for the entity is relevant
V. Ascertain the nature, timing and extent of resources necessary to perform the
engagement

None of the choices.

I,II,III, IV

I,IV,V

Correct!

I,II,III, IV, V
I,II, IV, V

Question 70
1 / 1 pts
The auditor should design the written audit program, so that:

Each account balance will be tested under either tests of controls or tests of transactions.

Substantive tests prior to the balance sheet date will be minimized.

Correct!

The audit procedures selected will achieve specific audit objectives.

All material transactions will be selected for substantive testing.

Question 71
1 / 1 pts
The element of the audit planning process most likely to be agreed upon with the client
before implementation of the audit strategy is the determination of the
Correct!

Timing of inventory observation procedures to be performed.

Pending legal matters to be included in the inquiry of the client's attorney.

Evidence to be gathered to provide a sufficient basis for the auditor's opinion


Procedures to be undertaken to discover litigation, claims, and assessments.

Question 72
1 / 1 pts
In developing the overall audit strategy, the focus of the engagement team’s efforts is
considered. Which of the following is not appropriately classified as a factor affecting the
focus of the team’s efforts?

Volume of transactions, which may determine whether it is more efficient for the auditor
to rely on internal control.

Setting materiality for planning purposes.

Correct!

The financial reporting framework on which the financial information to be audited has
been prepared, including any need for reconciliation to another reporting framework

Audit areas where there is a higher risk of material misstatement.

Question 73
1 / 1 pts
Adequate planning of the audit work helps the auditor of accomplishing the following
objectives, except:

The audit work is completed efficiently.

Identifying the areas that need a service of an expert.

Correct!
Gathering of all corroborating audit evidence.

Ensuring that appropriate attention is devoted to important areas of the audit.

Question 74
1 / 1 pts
Which of the following activities shall not be included in preplanning an audit?
Correct!

Determining the likelihood of issuing an unqualified audit opinion on the client’s financial
statements.

Investigating the client’s background.

Understanding the client’s reason for obtaining an audit.

Communicating with the prospective client’s prior auditor to inquire about any
disagreements with the client.

Question 75
1 / 1 pts
Which of the following statements is/are correct?
Statement I: The client should plan the audit work so that the audit will be performed in an
effective manner.
Statement II: The auditor should conduct the audit with an attitude of professional
skepticism.
Statement III: The auditor should develop and document an overall audit
plan describing the scope and conduct of the audit.
None of the choices.

I & III only

III only

Correct!

II & III only

II only

Question 76
1 / 1 pts
In developing an overall audit strategy, an auditor should consider:

Findings from substantive tests performed at interim dates.

Whether the allowance for sampling risk exceeds the achieved upper precision limit.

Correct!

Preliminary evaluations of materiality, audit risk, and internal control.

Whether the inquiry of the client's attorney identifies any litigation, claims, or
assessments not disclosed in the financial statements.
Question 77
1 / 1 pts
With respect to the auditor's planning of a year-end examination, which of the following
statements is always true?

The client's audit committee should not be told of the specific audit procedures that will
be performed.

An engagement should not be accepted after the fiscal year-end.

Correct!

It is an acceptable practice to carry out substantial parts of the examination at interim


dates.

An inventory count must be observed at the balance sheet date.

Question 78
1 / 1 pts
The early appointment of the independent auditor has many advantages to the auditor
and the client. Which of the following advantages is least likely to occur as a result of early
appointment of the auditor?
Correct!

The auditor will be able to complete the audit work in less time.

The auditor will be able to plan the audit work so that it may be done expeditiously.

The auditor will be able to perform the examination more efficiently and will be finished at
an early date after the year-end.
The auditor will be able to better plan for the observation of the physical inventories.

Question 79
1 / 1 pts
Which of the following procedures is not performed as a part of planning an audit
engagement?
Correct!

Test of controls

Designing an audit program

Performing analytical procedures

Reviewing working papers of the prior year

Question 80
0 / 1 pts
An audit program is ordinarily prepared for an audit engagement because:

It is required by generally accepted auditing standards.

It explains any weaknesses noted in the evaluation of the client’s existing internal control.

Correct Answer

It aids in instructing assistants in the work to be done.

You Answered
It documents the auditor’s understanding of the client’s internal control.

Question 81
0 / 1 pts
Which of the following statements is/are correct?
I. As defined in PSA 320, performance materiality means the amount or amounts set by
the auditor lower than materiality for the financial statements as a whole to reduce to an
appropriately low level the probability that the aggregate of uncorrected and undetected
misstatements exceeds materiality for the financial statements as a whole.
II. Based on PSA 320, when establishing the overall audit strategy, the auditor shall
determine materiality for the financial statements as a whole.
III. Based on PSA 320, If the auditor concludes that a lower materiality for the financial
statements as a whole than that initially determined is appropriate, the auditor shall
immediately revise performance materiality, and whether the nature, timing and extent of
the further audit procedures.
Correct Answer

I and II only

You Answered

II only

I and III only

None of the choices.

III only

Question 82
1 / 1 pts
Which of the following statements is not correct about materiality?
Correct!

An auditor considers materiality for planning purposes in terms of the largest aggregate
level of misstatements that could be material to any one of the financial statements.

An auditor’s consideration of materiality is influenced by the auditor’s perception of the


needs of a reasonable person who will rely on the financial statements.

Materiality judgments are made in light of surrounding circumstanced and necessarily


involve both quantitative and qualitative judgments.

The concept of materiality recognizes that some matters are important for fair
presentation of financial statements in conformity with GAAP, while other matters are
not important.

Question 83
1 / 1 pts
Which of the following statements is/are correct?
I. Overall materiality is the highest amount of misstatement/s that could be included in the
financial statements without affecting the economic decisions of a user.
II. Overall materiality is the materiality for the financial statements as a whole.

I only

Neither I nor II

II only
Correct!

Both I & II

Question 84
1 / 1 pts
Why should the auditor plan more work on individual accounts as lower acceptable levels
of both audit risk and materiality are established?

To find larger errors.

To decrease the risk of overreliance.

To increase the tolerable error in the accounts.

Correct!

To find smaller errors.

Question 85
1 / 1 pts
In considering materiality for planning purposes, Munda, auditor believes that
misstatements aggregating P60,000 would have material effect on an entity’s income
statement, but that misstatements would have to aggregate P40,000 to materially affect
the balance sheet. Ordinarily, it would be appropriate to design auditing procedures that
would be expected to detect misstatements that aggregate:

P50,000

Correct!

P40,000
P100,000

P60,000

Question 86
1 / 1 pts
. Which of the following statements concerning materiality thresholds is incorrect?
Correct!

In general, the more misstatements the auditor expects, the higher should be the
aggregate materiality threshold.

The smallest aggregate level of errors or fraud that could be considered material to any
one of the financial statements is referred to as a "materiality threshold."

Materiality thresholds may change between the planning and review stages of the audit.
These changes may be due to quantitative and/or qualitative factors.

Aggregate materiality thresholds are a function of the auditor's preliminary judgments


concerning audit risk.

Question 87
1 / 1 pts
Which of the following statements is true with regard to the relationship among audit risk,
audit evidence, and materiality?

Where inherent risk is high and control risk is low, the auditor may safely ignore inherent
risk.
Aggregate materiality thresholds should not change under conditions of changing risk
levels.

The lower the inherent risk and control risk, the lower the aggregate materiality
threshold.

Correct!

Under conditions of high inherent and control risk, the auditor should place more
emphasis on obtaining external evidence and should reduce reliance on internal evidence.

Question 88
1 / 1 pts
Which of the following would an auditor most likely use in determining the auditor’s
preliminary judgment about materiality?
Correct!

The entity’s annualized interim financial statements.

The contents of the management representation letter.

The anticipated sample size of the planned substantive tests.

The results of the internal control questionnaire.

Question 89
0 / 1 pts
Tolerable misstatement is the term used to indicate materiality at the:
company-wide level.

You Answered

assertion level

financial statement level.

Correct Answer

account balance level.

Question 90
0 / 1 pts
When comparing level of materiality used for planning purposes the level of materiality
used for evaluating evidence, one would most likely expect that

the level of materiality for planning purposes was based on assets while that used for
evaluation purposes was based on net income.

the level of materiality for planning and evaluation purposes is the same.

You Answered

the level of materiality for evaluation purposes is smaller.

Correct Answer

the level of materiality for planning purposes is smaller.

Question 91
1 / 1 pts
hich statement is incorrect regarding the discussion among the engagement team about
the susceptibility of the entity’s financial statements to material misstatements?
Correct!

All the team members should have a comprehensive knowledge of all aspects of the audit.

The objective of this discussion is for members of the engagement team to gain a better
understanding of the potential for material misstatements of the financial statements
resulting from fraud or error in the specific areas assigned to them, and to understand
how the results of the audit procedures that they perform may affect other aspects of the
audit.

The members of the engagement team should discuss the susceptibility of the entity’s
financial statements to material misstatements.

The discussion provides an opportunity for more experienced engagement team


members, including the engagement partner, to share their insights based on their
knowledge of the entity, and for the team members to exchange information about the
business risks.

Question 92
1 / 1 pts
To obtain an understanding of a continuing client’s business in planning an audit, an
auditor most likely will

re-evaluate the client’s internal control to see if there are significant changes.

read specialized industry journals and write-ups.

Correct!

review prior year working papers including the permanent file for the client.
perform test of details of material transactions and balances.

Question 93
1 / 1 pts
Audit procedures may be classified as risk assessment procedures and further audit
procedures. Which of the following best describes risk assessment procedures?
Correct!

These are procedures for obtaining an understanding of the entity and its environment,
including its internal control, to assess the risks of material misstatement at the financial
statement and assertion levels.

These procedures are used detect material misstatements at the assertion level.

These procedures include tests of details of classes of transactions, account balances, and
disclosures and analytical procedures.

These procedures test the operating effectiveness of controls in preventing, or detecting


and correcting, material misstatements at the assertion level.

Question 94
1 / 1 pts
The auditor’s understanding of the entity and its environment consists of an
understanding of the following aspects:
I. Industry, regulatory, and other external factors, including the applicable financial
reporting framework.
II. Nature of the entity, including the entity’s selection and application of accounting
policies.
III. Objectives and strategies and the related business risks that may result in a material
misstatement of the financial statements.
IV. Measurement and review of the entity’s financial performance.
V. Internal control
Correct!

I, II, III, IV and V

None of the choices.

I, II and III

I, II, III and IV

I, II, III and V

Question 95
1 / 1 pts
There is an inverse relationship that exist between the acceptable level of detection risk
and the

Risk of misapplying audit process

Risk of falling to discover material misstatement

Preliminary judgments about materiality levels

Correct!
Assurance provided by substantive tests

Question 96
0 / 1 pts
Analytical procedures used in planning an audit should focus on
Correct Answer

Enhancing the auditor’s understanding of the client’s business.

Reducing the scope of tests of controls and substantive tests.

You Answered

Assessing the adequacy of the available evidential matter.

Providing assurance that potential material misstatements will be identified.

Question 97
0 / 1 pts
The auditor should perform the following risk assessment procedures to obtain an
understanding of the entity and its environment, including its internal control, except:
Correct Answer

Inquiries of the entity’s external legal counsel or of valuation experts that the entity has
used.

Inquiries of management and others within the entity.

Analytical procedures.
Observation and inspection.

You Answered

All are risk assessment procedures.

Question 98
1 / 1 pts
Nature of an entity refers to

The operational approaches by which management intends to achieve its objectives.

The result of significant conditions, events, circumstances, actions or inactions that could
adversely affect the entity’s ability to achieve its objectives and execute its strategies, or
the setting of inappropriate objectives and strategies.

The overall plans for the entity.

Correct!

The entity’s operations, its ownership and governance, the types of investments that it is
making and plans to make, the way that the entity is structured and how it is financed.

Question 99
1 / 1 pts
In performing an audit of financial statements, the auditor should obtain a sufficient
knowledge of a client’s business and industry to

Make constructive suggestion concerning improvements causes the financial statements


taken as a whole to be materially misstated.
Evaluate whether the aggregation of known misstatements causes the financial
statements taken as a whole to be materially misstated.

Correct!

Understand the events and transactions that may have an effect on the client’s financial
statements.

Develop an attitude of professional skepticism concerning management’s financial


statements assertions.

Question 100
1 / 1 pts
Which of the following factors is not a good indicator of potential financial failure?

Client relies heavily on debt financing, especially by financing permanent assets with
short-term loans

Correct!

Client’s retained earnings were reduced by half as a result of a large dividend payout.

Client has had increasing net losses for several years.

Client is constantly short of cash and working capital.

Question 101
1 / 1 pts
Which of the following results from analytical procedures might indicate obsolete
inventory?
An increase in operating margin

A decline in days' sales in inventory

A decline in the gross margin ratio

Correct!

A decline in inventory turnover

Question 102
0 / 1 pts
Which of the following statements is/are correct
regarding obtaining an understanding of the entity and its environment?
I. The auditor’s primary consideration is whether the understanding that has been
obtained is sufficient to assess the risks of material misstatement of the financial
statements and to design and perform further audit procedures.
II. The depth of the overall understanding that is required by the auditor in performing the
audit is equal to that possessed by management in managing the entity.

Neither I nor II

You Answered

II only

Correct Answer

I only

Both I & II
Question 103
1 / 1 pts
Inquiries directed towards those charged with governance may most likely

Relate to changes in the entity’s marketing strategies, sales trends, or contractual


arrangements with its customers.

Correct!

Help the auditor understand the environment in which the financial statements are
prepared.

Relate to their activities concerning the design and effectiveness of the entity’s internal
control and whether management has satisfactorily responded to any findings from these
activities.

Help the auditor in evaluating the appropriateness of the selection and application of
certain accounting policies.

Question 104
1 / 1 pts
Analytical procedures performed in the planning stage of an audit suggest that several
accounts have unexpected relationships. The results of these procedures most likely
would indicate that:

Internal control activities are not operating effectively.

Tests of controls should be conducted.


Irregularities exist among the relevant account balances.

Correct!

Additional tests of details are required.

Question 105
0 / 1 pts
Which of the following procedures not normally performed as part of obtaining an
understanding of the client’s environment?
You Answered

Touring the client’s facilities.

Reading trade publications to gain a better understanding of the client’s industry.

Correct Answer

Confirming customer accounts receivable for existence and valuation.

Studying the internal controls over cash receipts and disbursements.

Question 106
1 / 1 pts
Internal controls are implemented to achieve the following purposes:
I. Financial statements are prepared according to the applicable financial reporting
framework. The financial reporting is reliable and relevance.
II. Business operations are efficient and effective.
III. The company complies with applicable law and regulations.

I, III
Correct!

I, II, II

None of the choices

I,II

Question 107
0 / 1 pts
In an auditor's consideration of internal control, the completion of a questionnaire is most
closely associated with which of the following?

Separation of duties.

Flowchart accuracy.

You Answered

Tests of controls.

Correct Answer

Understanding the system.

Question 108
1 / 1 pts
Which of the following statements about internal control is correct?
Correct!
The cost benefit relationship should be considered in designing internal controls.

Exceptionally strong control allows the auditor to eliminate substantive tests of details.

Establishing and maintaining internal control is the internal auditor's responsibility.

Properly maintained internal controls reasonably assure that collusion among employees
cannot occur.

Question 109
1 / 1 pts
Which of the following personnel department procedures reduces the risk of payroll fraud
and represents an appropriate responsibility of the department?

Authorizing overtime hours

Collection and retention of unclaimed paychecks

Correct!

Authorizing the addition or deletion of employees from the payroll

Distributing paychecks

Question 110
1 / 1 pts
The primary purpose of the auditor's consideration of internal control is to provide a basis
for
Determining whether procedures and records that are concerned with the safeguarding
of assets are reliable.

Correct!

Determining the nature, timing, and extent of audit tests to be applied.

The expression of an opinion.

Constructive suggestions to clients concerning deficiencies in internal control.

Question 111
1 / 1 pts
Is an audit procedure designed to evaluate the operating effectiveness of controls in
preventing, or detecting and correcting, material misstatements at the assertion level.

Substantive Testing

Correct!

Test of Controls

Test for effectiveness

Evaluative Test

Question 112
1 / 1 pts
An auditor should consider the competence of a client's employees because their
competence bears directly and importantly on the
Correct!

Achievement of the objectives of internal control.

Timing of the tests to be performed.

Comparison of recorded accountability with assets on hand.

Cost benefit relationship of internal control.

Question 113
1 / 1 pts
Which of the following procedures most likely would provide an auditor with evidence
about whether an entity's internal control is suitably designed to prevent or detect
material misstatements?

Scanning the journals produced by the internal control system

Performing analytical procedures using data aggregated at a high level

Vouching a sample of transactions directly related to the controls

Correct!

Observing the entity's personnel applying the controls

Question 114
1 / 1 pts
An auditor should obtain sufficient knowledge of an entity’s information system, including
the related business processes relevant to financial reporting, to understand the

Policies used to detect the concealment of fraud.

Safeguards used to limit access to computer facilities.

Procedures used to assure proper authorization of transactions.

Correct!

Process used to prepare significant accounting estimates.

Question 115
0 / 1 pts
An entity's information system infrastructure refers to
You Answered

Software

Data provided by the system

Programmers

Correct Answer

Hardware components

Question 116
1 / 1 pts
Which of the following is likely to be of least importance to an auditor when assessing
control
risk in a company that processes data by computer?

The segregation of duties within the computer department.

Correct!

The cost-benefit ratio of data processing operations.

The documentation maintained for accounting applications.

The control over source documents.

Question 117
0 / 1 pts
Information and communication includes all of the following except

Properly presenting transactions and related disclosures in the financial statements

Correct Answer

Communicating price changes to customers

Determining the time period in which transactions occurred

You Answered

Identifying and recording all valid transactions


Question 118
1 / 1 pts
The purpose of tests of controls is to provide reasonable assurance that

The extent of substantive testing is minimized.

The auditor has an understanding of the control environment.

Errors and irregularities are prevented or detected in a timely manner.

Correct!

Evidence will be obtained to determine an assessed level of control risk.

Question 119
1 / 1 pts
An advantage of using systems flowcharts to document information about internal control
instead of using internal control questionnaires is that systems flowcharts
Correct!

Provide a visual depiction of clients' activities

Reduce the need to observe clients' employees performing routine tasks

Identify whether segregation of duties prevent collusion

Indicate whether controls are operating effectively


Question 120
1 / 1 pts
The documentation of an auditor's understanding of internal controls

Must include flowcharts

Is optional

Must be exclusively in either narrative, questionnaire, or flowchart form

Correct!

Can use any combination of narratives, questionnaire, or flowcharts

Question 121
0 / 1 pts
An auditor should consider two key issues when obtaining an understanding of a client’s
internal controls. These issues are
You Answered

The frequency and effectiveness of the controls.

The implementation and efficiency of the controls.

The effectiveness and efficiency of the controls

Correct Answer

The design and implementation of the controls.


Question 122
1 / 1 pts
This is the tone or overall attitude of the organization regarding internal controls. This is
integrated on the actions, and policies coming from the board of directors, top
management and owners of organizations.

Information and Communication

Control Activities

Risk Assessment

Monitoring

Correct!

Control Environment

Question 123
1 / 1 pts
Which of the following statements is/are correct?

1. Effectiveness of internal control is the responsibility of the auditor.


2. Internal controls are categorized according to their function namely:
preventive, detective and corrective.

Both I & II

Neither I nor II
I only

Correct!

II only

Question 124
0 / 1 pts
Which of the following control activities in an entity’s revenue/receipt cycle would
provide reasonable assurance that all billed sales are correctly posted to the accounts
receivable ledger?

Each shipment of goods on credit is supported by a prenumbered sales invoice.

You Answered

Daily sales summaries are compared to daily postings to the accounts receivable ledger.

Each sales invoice is supported by a prenumbered shipping document.

Correct Answer

Daily sales summaries are compared to daily postings to the accounts receivable ledger.

Question 125
1 / 1 pts
An auditor would most likely be concerned with internal control policies and procedures
that provide reasonable assurance about the
Correct!

Entity's ability to accurately process and summarize financial data.


Appropriate prices that the entity should charge for its products.

Efficiency of management's decision-making process.

Safekeeping of entity’s assets and other resources.

Question 126
0 / 1 pts
This component of internal control evaluates whether the components of internal
controls including the subcomponents are present and functioning accordingly.
You Answered

Information and Communication

Correct Answer

Monitoring

Control Environment

Control Activities

Risk Assessment

Question 127
1 / 1 pts
Which of the following would be a preventive control?

Preparation of bank reconciliation


Reconciling the accounts receivable subsidiary file with the control account.

The use of batch totals.

Correct!

Requirement that two persons open a mail.

Question 128
1 / 1 pts
Internal control procedures are not designed to provide reasonable assurance that

The recorded accountability for assets is compared with the existing assets at reasonable
intervals

Transactions are executed in accordance with management's authorization.

Access to assets is permitted only in accordance with management's authorization.

Correct!

Irregularities will be eliminated.

Question 129
1 / 1 pts
Proper segregation of functional responsibilities calls for separation of the functions of
Correct!

Authorization, custody, and recording.


Authorization, payment, and recording.

Authorization, execution, and payment.

Custody, execution, and reporting.

Question 130
1 / 1 pts
In general, a material weakness in internal control may be defined as a condition in which
material errors or irregularities may occur and not be detected within a timely period by

Outside consultants who issue a special-purpose report on internal control structure.

An independent auditor during tests of controls.

Correct!

Employees in the normal course of performing their assigned functions.

Management when reviewing interim financial statements and reconciling account


balances.

Question 1
2 / 2 pts
An auditor is testing credit authorization procedures by examining sales invoices for
credit approval by the credit department. The procedures will be considered to be
working adequately if 96% of all sales invoices either indicate approval or are cash sales.
The auditor selects a random sample of 100 invoices. In this situation, which of the
following outcomes illustrates underassessment or the risk of assessing control risk too
low?
Correct!

The auditor finds no deviations and concludes that procedures work adequately. The true
population deviation rate is 5%.

The auditor finds no deviations and concludes that the procedures work adequately. The
true population deviation rate is 2%.

The auditor finds five deviations and concludes that procedures work inadequately. The
actual population deviation rate is 2%.

The auditor finds five deviations and concludes that procedures work inadequately. The
true population deviation rate is 6%.

Question 2
0 / 2 pts
Which of the following statements is correct concerning statistical sampling in tests of
controls?

As the population size doubles, the sample size also should double.

The expected population deviation rate has little or no effect on determining sample size
except for very small populations.

Correct Answer

The population size has little or no effect on determining sample size except for very small
populations.

You Answered
For a given tolerable rate, a larger sample size should be selected as the expected
population deviation rate decreases.

Question 3
0 / 2 pts
Which of the following would most likely be detected by an auditor's review of a client's
sales cutoff?
Correct Answer

Unrecorded sales for the year

You Answered

Lapping of year end accounts receivable.

Excessive sales discounts.

Unauthorized goods returned for credit.

Question 4
2 / 2 pts
Which of the following audit procedures is least likely to detect an unrecorded liability?

Reading the minutes of board of directors' meetings

Mailing a standard bank confirmation form.

Analysis and recalculation of interest expense.

Correct!
Analysis and recalculation of depreciation expense.

Question 5
0 / 2 pts
Which of the following is the least persuasive type of audit evidence?
You Answered

Correspondence between auditor and vendors.

Computations made by the auditor.

Documents mailed by outsiders to the auditor.

Correct Answer

Copies of sales invoices inspected by the auditor.

Question 6
2 / 2 pts
Auditors may use positive and/or negative forms of confirmation requests for accounts
receivable. An auditor most likely will use

A combination of the two forms, with the positive form used for trade receivables and the
negative form for other receivables.

The positive form when the control structure related to receivables are satisfactory, and
the negative form when controls are unsatisfactory.

The positive form to confirm all balances, regardless of size.


Correct!

A combination of the two forms, with the positive form used for large balances and the
negative form for small balances.

Question 7
0 / 2 pts
The risk of incorrect acceptance and the risk of assessing control risk too low relate to the

Preliminary estimates of materiality levels.

You Answered

Allowable risk of tolerable misstatement.

Correct Answer

Effectiveness of the audit.

Efficiency of the audit.

Question 8
2 / 2 pts
The use of prenumbered invoices, then accounting for their numeric sequence, would
prove which of the following assertion?
Correct!

Completeness

Cutoff
Occurrence

Accuracy

Question 9
2 / 2 pts
Which of the following sampling methods is most useful to auditors when testing for
internal control effectiveness?
Correct!

Attribute sampling

Stratified random sampling

Unrestricted random sampling with replacement

Variables sampling

Question 10
2 / 2 pts
Which of the following types of procedures will aid the auditor in obtaining evidence
regarding the mathematical accuracy of accounting records and other information?

Confirmation

Correct!

Recalculation
Inspection

Analytical Procedures

Question 11
2 / 2 pts
When performing a test of a control with respect to control over cash receipts, an auditor
may use a systematic sampling technique with a start at any randomly selected item. The
biggest disadvantage of this type of sampling is that the items in the population
Correct!

May occur in a systematic pattern, thus destroying the sample randomness.

May systematically occur more than once in the sample.

Must be recorded in a systematic pattern before the sample can be drawn.

Must be systematically replaced in the population after sampling.

Question 12
2 / 2 pts
You are part of the engagement team of Frontliners Auditing Firm which was assigned as
auditor of COVID Company. During the course of the audit, you anticipate reliance on the
client’s internal control. Thus, you would

Reduce the extent of substantive tests as a result of reliance to internal control.

Perform tests of controls and increase the amount of substantive tests.


No longer perform tests of controls and proceed to substantive tests.

Correct!

Test controls and use the results of testing as a basis for determining the nature, extent
and timing of substantive tests.

Question 13
2 / 2 pts
Which of the following statements is/are correct?
I. The reliability of evidence is influence by its nature and materiality.
II. Sufficiency is the measure of the quantity of evidence which the standards require.

I only

Correct!

Neither I nor II

II only

Both I & II

Question 14
0 / 2 pts
An advantage of using statistical over non-statistical sampling methods in tests of controls
is that the statistical methods
You Answered
Can more easily convert the sample into a dual-purpose test useful for substantive
testing.

Eliminate the need to use judgment in determining appropriate sample sizes.

Afford greater assurance than a non-statistical sample of equal size.

Correct Answer

Provide an objective basis for quantitatively evaluating sample risk.

Question 15
2 / 2 pts
An auditor performs a test to determine whether all merchandise for which the client was
billed was received. The population for this test consists of all

Canceled checks.

Correct!

Vendors' invoices.

Receiving reports.

Merchandise received.

Question 16
2 / 2 pts
In determining the existence of accounts receivable, which of the following would the
auditor consider most reliable?
Correct!

Confirmation replies received directly from customers.

Direct telephone communication between auditor and debtor.

Documents that supports the accounts receivable balance.

Credits to accounts receivable from the cash receipts book after the close of business at
year-end.

Question 17
0 / 2 pts
While performing a test of details during an audit, an auditor determined that the sample
results supported the conclusion that the recorded account balance was materially
misstated. It was, in fact, not materially misstated. This situation illustrates the risk of

Assessing control risk too low.

Assessing control risk too high.

Correct Answer

Incorrect rejection.

You Answered

Incorrect acceptance.

Question 18
2 / 2 pts
How would increases in tolerable misstatement and assessed level of control risk affect
the sample size in a substantive test of details?

Decrease sample size; Decrease sample size

Correct!

Decrease sample size; Increase sample size

Increase sample size; Increase sample size

Increase sample size; Decrease sample size

Question 19
2 / 2 pts
Substantive audit procedures

May be eliminated under certain circumstances.

Are designed to detect deviations from prescribed entity procedures.

Correct!

May be either tests of transactions, tests of balances, or analytical procedures.

Will increase proportionately with the auditor's assessment of control risk.


Question 20
2 / 2 pts
An auditor is testing internal control procedures that are evidenced on an entity’s
vouchers by matching random numbers with voucher numbers. If a random number
matches the number of a voided voucher, that voucher ordinarily should be replaced by
another voucher in the random sample if the voucher

Cannot be located.

Represents an immaterial peso amount.

Correct!

Has been properly voided.

Constitutes a deviation.

Question 21
2 / 2 pts
The tolerable deviation rate for a control test is generally
Correct!

Higher than the expected deviation rate in the related accounting records.

Identical to the expected deviation rate in the related accounting records

Unrelated to the expected deviation rate in the related accounting records.

Lower than the expected deviation rate in the related accounting records.
Question 22
2 / 2 pts
A number of factors influences the sample size for a substantive test of details of an
account balance. All other factors being equal, which of the following would lead to a
larger sample size?

Lowering of assessed inherent risk through use of analytical review procedures

Correct!

Smaller measure of tolerable error

Smaller expected frequency of errors

Lowering of assessed level of control risk

Question 23
2 / 2 pts
Assertions used by the auditor fall into three categories. Existence is included under

Assertions about presentation and disclosure.

Correct!

Assertions about account balances at the period end.

All of the choices.

Assertions about classes of transactions and events for the period under audit.
Question 24
2 / 2 pts
Which of the following best illustrates the concept of sampling risk?

The documents related to the chosen sample may not be available for inspection.

Correct!

A randomly chosen sample may not be representative of the population as a whole on the
characteristic of interest.

An auditor may select audit procedures that are not appropriate to achieve the specific
objective.

An auditor may fail to recognize errors in the documents examined for the chosen sample.

Question 25
0 / 2 pts
Tests of controls are designed to obtain evidence to support the auditor’s assessment of
control risk

at the lowest level

at zero level

You Answered

at maximum level

Correct Answer
at less than high level

Question 26
2 / 2 pts
A bank auditor is interested in estimating the average account balance of its depositors
based on a sample. This substantive test is an example of

Discovery sampling.

Sequence sampling.

Attribute sampling.

Correct!

Variables sampling.

Question 27
2 / 2 pts
In determining the sample size for a test of controls, an auditor should consider the likely
rate of deviations, the allowable risk of assessing control risk too low, and the

Risk of incorrect acceptance.

Correct!

Tolerable deviation rate.

Nature and cause of deviations.


Population size.

Question 28
2 / 2 pts
Which of the following audit procedures would be most appropriate to address the
existence assertion for sales?

Perform analytical procedures

Correct!

Confirm receivables balances

Confirm cash deposits in banks

Review collectibility

Question 29
0 / 2 pts
The auditor should perform tests of controls on

Relevant controls that were reviewed

You Answered

Operating effectiveness of relevant controls which represent material weaknesses.

Relevant controls that have a material effect on the financial statements.

Correct Answer
Relevant controls when the auditor intends to rely on such controls in determining the
nature, timing and extent of substantive procedures.

Question 30
0 / 2 pts
Which of the following factors is most important in determining the appropriateness of
audit evidence?

The objectivity of the auditor gathering the evidence.

Correct Answer

The reliability of the evidence in meeting the audit objective.

You Answered

The independence of the source of evidence.

The quantity of the evidence obtained.

Question 1
1 / 1 pts
Elijah, CPA decided to test the computer information system by testing the input and
output of the computer instead of testing the computer program itself. Which of the
following is a disadvantage of this kind of testing?
Correct!

It will not detect program errors which do not show up in the output.

It will make the auditor’s opinion unqualified.


It will not give auditors evidence at all.

It will be able to detect fraud more effectively.

Question 2
1 / 1 pts
Substantive analytical procedures are generally more applicable to:
Correct!

large volumes of transactions that tend to be predictable over time.

large volumes of transactions.

small volumes of transactions.

small volumes of transactions that tend to be predictable over time.

Question 3
0 / 1 pts
Which of the following statements is/are correct?
Statement I. The quantity of audit evidence needed is affected by the risk of misstatement
and also by the quality of such audit evidence.
Statement II. The reliability of audit evidence is influenced by its source and by its nature
and is dependent on the individual circumstances under which it is obtained.
You Answered

II only
I only

Neither I nor II

Correct Answer

both I and II

Question 4
0 / 1 pts
An auditor want to assess the risk in a computerized environment. Under these
circumstances on which of the following activities would the auditor initially focus?

Errors and fraud

Correct Answer

Controls over outputs

You Answered

General controls

Application controls

Question 5
1 / 1 pts
On the four situations presented below. Which circumstance constitutes significant
deficiency in internal control?
Correct!
Supervision of programmers to the computer operators.

System analysts should not be able to enter transactions in the programs they created.

Computer operators are not given access to the software support documentation.

System analysts should not be able to operate computers.

Question 6
1 / 1 pts
Benjamin, CPA decided to use stratification in his random sampling. Which of the
following is the BEST reason why Benjamin choose to use stratification rather than
unstratified random sampling?
Correct!

Benjamin wants to reduce as much as possible the degree of variability in the overall
population.

Benjamin wants to focus on the larger items in the population

Benjamin wants a larger sample size.

Benjamin wants to give every sampling unit an equal chance of being included in the
sample.

Question 7
1 / 1 pts
Consists of seeking information from knowledgeable persons, both financial and
nonfinancial, within the entity or outside the entity.
Inspection of records or documents

Correct!

Inquiry

Observation

Inspection of tangible assets

Question 8
0 / 1 pts
. Example of factors influencing sample size for test of controls includes:
I. Other things being constant, the greater the reliance the auditor places on the operating
effectiveness of internal controls in the risk assessment, the greater is the extent of the
auditor’s test of controls, and therefore, the sample size is increased.
II. The higher the tolerable rate of deviation, the lower the sample size will be. The
tolerable rate of deviation is a rate of deviation from prescribed internal controls
procedures set by the auditor in respect of which the auditor seeks to obtain an
appropriate level of assurance that the rate of deviation set by the auditor is not exceeded
by the actual rate of deviation in the population. Accordingly, the tolerable rate of
deviation is the rate of deviation that an auditor is willing to accept without modifying the
initial assessed level of control risk.
III. An increase in the estimate of the deviation rate in the entire population the larger the
sample size needs to be so that the sample could reasonably reflect the actual rate of
deviation in the population.
IV. An increased in the level of assurance by the auditor that the tolerable rate of
deviation is not exceeded by the actual rate of deviation in the population, the larger the
sample size will be.
V. An increase in the number of sampling units in the population have little to no effect on
sample size.

None of the choices


I, III, V

Correct Answer

I, II, III, IV, V

I, II, III, IV

You Answered

II, IV, V

Question 9
0 / 1 pts
Which of the following controls ensure that the results of computer processing are
accurate, complete, and properly distributed?
You Answered

General Controls

Correct Answer

Output Controls

Processing Controls

Input Controls

Question 10
1 / 1 pts
The most likely concern of an auditor regarding controls in a distributed data processing
system is?

Data recovery controls

Correct!

Access controls

Systems documentation controls

Control over hardware

Question 11
1 / 1 pts
Test of Controls Sampling Risks includes:
I. Risk of assessing control risk too high (alpha risk, type I error)
II. Risk of assessing control risk too low (beta risk, type II error)

None of the Choices

II

Correct!

Both
Question 12
0 / 1 pts
Reconciliations is:
You Answered

an audit procedure that involves tracing an accounting information from source document
to another subsequent documents like journal or general ledger.

Correct Answer

an audit procedure establishing agreement between separate sources of information such


accounting records with bank statements.

None of the choices

an audit procedure that involves checking the mathematical accuracy of document and
records.

Question 13
1 / 1 pts
Flora, CPA is performing tests of controls over nontrade purchases. Flora would like to
find out whether the company’s control of reviewing nontrade purchase before being
entered in the accounting system is operating effectively. Accordingly, Flora determined
that the supervisor stamped nontrade purchase after it has been reviewed. In this
situation, Flora tested fifty (50) nontrade purchase orders to identify they have been
stamped or not. She also set the tolerable rate at 7%, the expected population at 5% and
the allowance for sampling risk at 2%. Afterwards, the results showed that three nontrade
purchase orders where not stamped and reviewed by the supervisor before it was
recorded. In this situation, what will be Flora’s evaluation of the sampling results for this
particular control?
Correct!

Flora needs to modify the planned assessed level of control risk because the sample
deviation rate plus the allowance for sampling risk exceeds the tolerable rate.
Flora needs to modify the initial assessed level of control risk because the tolerable
deviation rate plus the allowance for sampling risk exceeds the expected population
deviation rate.

Flora shall accept the sample results as support for the initial assessed level of control risk
because the tolerable deviation rate less the allowance for sampling risk equals the
expected population deviation rate.

Flora shall accept the sample results as support for the initial assessed level of control risk
because the sample deviation rate plus the allowance for sampling risk exceeds the
tolerable rate.

Question 14
1 / 1 pts
Which statement is incorrect regarding the evaluation of general CIS controls and CIS
application controls?

Manual procedures exercised by users may provide effective control at the application
level.

Correct!

Weaknesses in general CIS controls cannot preclude testing certain CIS application
controls.

The general CIS controls may have a pervasive effect on the processing of transactions in
application systems.

If general CIS controls are not effective, there may be a risk that misstatements might
occur and go undetected in the application systems.
Question 15
1 / 1 pts
Vouching is:

an audit procedure that involves the auditor seeing something or observing an item
firsthand.

An audit procedure performed to see if an activity will yield the same results if done
similarly by the auditor.

Correct!

An audit procedure that begins tracking an information from one document like general
ledger or journal back into its underlying source documents.

None of the choices

Question 16
1 / 1 pts
Assertions about account balances at the period-end include valuation and allocation,
which means that

The entity holds or controls the rights to assets, and liabilities are the obligations of the
entity.

All assets, liabilities and equity interests that should have been recorded have been
recorded.

Assets, liabilities and equity interest exist.

Correct!
Assets, liabilities and equity interests are included in the financial statements at
appropriate amounts and any resulting valuation or allocation adjustments are
appropriately recorded.

Question 17
0 / 1 pts
Which of the following does not involve audit sampling?
Correct Answer

Substantive analytical procedure

Test of details of transactions applied as a substantive test

You Answered

Test of details of transactions applied as a test of control

Test of details of balances

Question 18
1 / 1 pts
Which of the following statistical selection techniques is least desirable for use by an
auditor?

Stratified selection

Systematic selection

Correct!
Block selection

Sequential selection

Question 19
0 / 1 pts
Examples of Statistical Sampling includes:
I. Random Sampling
II. Systematic Selection
III. Monetary Unit Sampling

You Answered

I, II

None of the choices

Correct Answer

I, II, III

Question 20
1 / 1 pts
The process of vouching helps establish that all recorded transactions are

Complete
Presented properly

Recorded

Correct!

Valid

Question 21
1 / 1 pts
Analytical procedures include the consideration of comparisons of the entity’s financial
information with, for example:
I. Comparable information for prior periods.
II. Anticipated results of the entity, such as budgets or forecasts, or expectations of the
auditor, such as an estimation of depreciation.
III. Similar industry information, such as a comparison of the entity’s ratio of sales to
accounts receivable with industry averages or with other entities of comparable size in
the same industry.
Correct!

I, II, III

I, II

None of the choices


Question 22
1 / 1 pts
Which of the following management assertions is an auditor most likely testing if the audit
objective states that all inventory on hand is reflected in the ending inventory balance?
Correct!

Inventory is complete.

The entity has rights to the inventory.

Inventory is properly presented in the financial statements.

Inventory is properly valued.

Question 23
1 / 1 pts
Which of the following methods of testing application controls utilizes a generalized audit
software package prepared by the auditors?

Test data approach

Integrated testing facility approach

Exception report tests

Correct!

Parallel simulation
Question 24
0 / 1 pts
There are various approaches to test general controls, some are:
I. Auditors may test the program development and maintenance by obtaining and
examining documentation or plans related to program development. Also, the auditor may
inquire personnel and review minutes of the meeting of personnel charged with computer
information systems responsibilities.
II. To test system software the auditor may inquire with appropriate personnel or examine
company software manual.
III. For testing operations, the auditor may observe the separation of duties of systems
analyst, programmers and computer operators or inquiry of operators about their
adherence to policies and operation manuals.
IV. The auditor tests physical security relating computer information systems by
examining documentary evidence about the use of backup or inquire about the security
over physical access to the information technology facility.
V. Testing the controls over access to programs includes an examination of documentary
evidence related to authorization of access or, if any, review log access including access
violations
Correct Answer

None of the choices

I, II

I, II, III

You Answered

I, II, III, IV, V

I, II, III, IV
Question 25
1 / 1 pts
Which of the following is a disadvantage of the integrated test facility approach?
Correct!

Removing the fictitious transactions from the system is somewhat difficult and, if not
done carefully, may contaminate the client's files.

In establishing fictitious entities, the auditor may be compromising audit independence.

ITF is simply an automated version of auditing "around" the computer.

The auditor may not always have a current copy of the authorized version of the client's
program.

Question 26
0 / 1 pts
Which of the following statements about test data is/are correct?
Statement I: The test data must consist of all possible valid and invalid conditions.
Statement II: When using test data method, the program tested is different from the
program used throughout the year by the client.

You Answered

II

Both
Correct Answer

Neither I nor II

Question 27
0 / 1 pts
All the following are examples of computer-assisted audit technique, except

Audit modules.

You Answered

Audit hooks.

Correct Answer

Operating systems.

Integrated test data.

Question 28
1 / 1 pts
Substantive procedures is comprising of:
I. Test of details
II. Substantive Analytical Procedures

II

Correct!

I and II
None of the above

Question 29
1 / 1 pts
The output of a parallel simulation should always be.
Correct!

Compared with actual results manually.

Reconciled to actual processing output.

Compared with actual results using a comparison program.

Printed on a report.

Question 30
1 / 1 pts
Which of the following statements related to sampling is incorrect?

Population is the entire set of data from which a sample is selected and about which the
auditor wishes to draw conclusions.

Random sample is a sample in which every possible combination of items in the population
has an equal chance of constituting the sample.

Correct!
A statistical sample in which the characteristics in the sample are the same as those of the
population.

Sampling unit pertains to individual items constituting a population.

Question 31
1 / 1 pts
Which of the following statements is/are correct?
Statement I: The tolerable rate of deviations for a test of a control is generally lower than
the expected rate of errors in the related accounting records.
Statement II: In determining the sample size for a test of controls, an auditor should
consider the expected deviation rate, desired confidence level, and the tolerable deviation
rate.

Both I & II

Neither I nor II

I only

Correct!

II only

Question 32
1 / 1 pts
The auditor notices that a client’s cash-basis financial statements are prepared with
accrual basis financial titles. This situation bears on which financial statement assertion?

Valuation or allocation
Completeness

Correct!

Presentation and disclosure

Rights and obligations

Question 33
1 / 1 pts
In sampling plan for test of details, an auditor must generally consider each of the
following except

Variation within the population

Correct!

Population size

Acceptable risk of incorrect acceptance

Tolerable error

Question 34
1 / 1 pts
A number of factors influence the sample size for a substantive test of details of an
account balance. All other factors being equal, which of the following would lead to a
larger sample size?
Smaller expected frequency of errors

Correct!

Smaller measure of tolerable misstatement

Greater reliance on analytical procedures

Greater reliance on internal controls

Question 35
1 / 1 pts
“Analytical procedures” means

evaluations of financial information through analysis of unlikely relationships among both


financial and non-financial data.

evaluations of financial information through analysis of plausible relationships among


non-financial data only.

Correct!

evaluations of financial information through analysis of plausible relationships among


both financial and non-financial data.

evaluations of financial information through analysis of plausible relationships among


financial data only.

Question 36
1 / 1 pts
Albert, an auditor, was assigned to identify the management’s assertions about class of
transactions, events, related disclosures. Accordingly, which of the following is NOT an
assertion about class of transactions?

Occurrence

Cutoff

Correct!

Consistency

Completeness

Question 37
0 / 1 pts
General controls relate to controls over:
I. program development and maintenance
II. systems software support
III. operations
IV. physical system security
V. access to programs

I, III, V

Correct Answer

I, II, III, IV, V

I, II, IV
You Answered

None of the choices

I, II, III, IV

Question 38
1 / 1 pts
Which of the following combinations results in a larger sample size?
Correct!

Increase the desired confidence level and decrease the tolerable deviation rate

Decrease the desired confidence level and increase the expected deviation rate

Decrease the desired confidence level and decrease the tolerable deviation rate

Increase the tolerable deviation rate and increase the expected deviation rate

Question 39
1 / 1 pts
Daniel, CPA, as part of the audit engagement, will be testing the general and application
controls of Agustin Company which uses a batch system. Which of the following is TRUE
concerning batch processing of transactions?

Transactions are processed in chronological order, regardless of type.

It cannot be used for large network of systems.


It used only in non-database applications.

Correct!

It is more likely to result in an easy-to-follow audit trail than is online transaction


processing.

Question 40
1 / 1 pts
All of the following are included in the audit documentation, except

The auditor’s conclusion as to whether uncorrected misstatements are material,


individually, or in aggregate, and the basis for that conclusion

The nature, timing, and extent of the further audit procedures performed.

Correct!

The amount below which misstatement would be regarded as clearly not trivial

All misstatements accumulated during the audit and whether they have been corrected

Question 41
1 / 1 pts
The maximum error in a population that the auditor is willing to accept

Anomalous error

Correct!
Tolerable error

Sample error

Expected Population Error

Question 42
1 / 1 pts
For control purposes, which of the following should be organizationally segregated from
the computer operations function?
Correct!

Systems development

Data conversion

Surveillance of CRT messages

Minor maintenance according to a schedule

Question 43
1 / 1 pts
Regardless of the assessed risks of material misstatement, the auditor is required to
design and perform substantive procedures for each material class of transactions,
account balance, and disclosure. This reflects the facts that:
I. The auditor’s assessment of risk is judgmental and so may not identify all risks of
material misstatement
II. There are inherent limitations to internal control
II

Correct!

I, II

None of the choices

Question 44
0 / 1 pts
A main problem present in a CIS environment is that incompatible duties may be
performed by the one person. One compensating control for this is the use of

Third party observer

Self-checking digits

You Answered

Access controls

Correct Answer

A computer log

Question 45
1 / 1 pts
Tests of controls may include the following, except:
Inspection of documentary support to transactions evidencing authorization

Correct!

Analytical procedures involving comparison of operating expenses with budget amount.

Reperformance of internal control procedures.

Inquiries about, and observation of, internal controls which leave no audit trail.

Question 46
0 / 1 pts
John Paul, CPA is performing an audit examination of Malfoy, Inc. He has already done the
work necessary to gain a general understanding of the company’s internal control. He is
now looking at the internal control policies and procedures within the accounting system
specifically for accounts receivable. He has decided to perform test of controls in this
area. Which of the following is least likely to be true?

He hopes to reduce the necessary amount of substantive testing.

You Answered

He hopes to reduce overall audit time.

The internal control system appears to be well designed.


Correct Answer

The system contains an excess number of possible problems so that testing is required.

Question 47
1 / 1 pts
Which of the following controls most likely would assure that an entity can reconstruct its
financial records?

Personnel who are independent of data input perform parallel simulations.

Hardware controls are built into the computer by the computer manufacturer.

System flowcharts provide accurate descriptions of input and output operations.

Correct!

Backup diskettes or tapes of files are stored away from originals.

Question 48
0 / 1 pts
Which of the following statements is/are correct?
Statement I: Those controls subject to testing by performing inquiry combined with
inspection or reperformance ordinarily provide more assurance than those controls for
which the audit evidence consists solely of inquiry and observation.
Statement II: If the auditor plans to rely on controls that have not changed since they were
last tested, the auditor should test the operating effectiveness of such controls at least
once in every third audit.
You Answered

Neither I nor II
I only

Correct Answer

both I and II

II only

Question 49
1 / 1 pts
Audit sampling involves the
Correct!

Application of audit procedures to less than 100% of items within a class of transactions
or an account balance such that all items have a chance of selection

Selection of all items over a certain amount

Application of audit procedures to all items over a certain amount and those that are
unusual or have a history of error

Application of audit procedures to all items that comprise a class of transactions or an


account balance

Question 50
1 / 1 pts
Application control is normally divided into the following categories:
I. Input controls
II. Processing controls
III. Output controls
Correct!

I, II, III

None of the choices

I, II

Question 51
1 / 1 pts
Test of controls is performed when:

All of the choices

the auditor believes that the internal control is not operating effectively

Correct!

the auditor plans to rely on the operating effectiveness of internal controls.

the auditor has no plans to rely on the operating effectiveness of internal controls.

Question 52
1 / 1 pts
Which of the following most likely represents a significant deficiency in the internal
control structure?
Correct!

The systems programmer designs systems for computerized applications and maintains
output controls.

The systems analyst review applications of data processing and maintains systems
documentation.

The control clerk establishes control over data received by the EDP department and
reconciles control totals after processing.

The accounts payable clerk prepares data for computer processing and enters the data
into the computer.

Question 53
1 / 1 pts
Tracing copies of sales invoices to shipping documents will provide evidence that all

Shipments to customers were billed.

Correct!

Billed sales were shipped.

Shipments to customers were recorded as receivables.

Debits to the subsidiary accounts receivable ledger are for sales shipped.

Question 54
1 / 1 pts
The nature of the risks and the internal control characteristics in computer information
systems environments generally include the following:
I. Lack of transaction trails
II. Uniform processing of transactions
III. Lack of segregation of functions
IV. Potential for errors and irregularities
V. Errors and irregularities that will not be observed due to less human involvement in
handling transactions
VI. Initiation or execution of transactions
VII. The dependence of other controls over computer processing
VIII. Potential for increased management supervision
Correct!

I, II, III, IV, V, VI, VII, VIII

None of the choices

II, IV, VI, VIII

I, III, V, VII

Question 55
1 / 1 pts
Which of the following characteristics distinguishes computer processing from manual
processing?

The potential for systematic error is ordinarily greater in manual processing than in
computerized processing.
Most computer systems are designed so that transaction trails useful for audit do not
exist.

Errors or irregularities in computer processing will be detected soon after their


occurrences.

Correct!

Computer processing virtually eliminates the occurrence of computational error normally


associated with manual processing.

Question 56
1 / 1 pts
The auditor faces a risk that the examination will not detect material misstatements in the
financial statements. In regard to minimizing this risk, the auditor primarily relies on:

Statistical analysis.

Tests of controls.

Internal control.

Correct!

Substantive tests.
Question 57
1 / 1 pts
Segregation of incompatible duties is differently defined in a manual system since the
computer programs can do multitasking. To offset the negative effects for the lack of
segregation of duties, how can a computer programs be modified?

The computer system should be accessible to various competent partners so they can
check on each other’s work.

Run complete parallel manual and automated accounting systems for a crisscross check
on input and output.

Correct!

Effective built- in controls for both the computer software and hardware to reduce the
risk of unauthorized access and manipulation.

The computer system should be under the control of the user department.

Question 58
1 / 1 pts
The likelihood of assessing control risk too high is the risk that the sample selected to test
controls

Does not support the tolerable error for some or all of management’s assertions.

Contains proportionately fewer monetary errors or deviations from prescribed controls


than exist in the balance or class as a whole.

Contains misstatements that could be material to the financial statements when


aggregated with misstatements in other account balances or transactions classes.
Correct!

Does not support the auditor’s planned assessed level of control risk when the true
operating effectiveness of the control structure justifies such an assessment.

Question 59
1 / 1 pts
Which of the following tasks could not be performed when using a generalized audit
software package?

Summarizing inventory turnover statistics for obsolescence analysis.

Comparison of inventory test counts with perpetual records.

Selecting inventory items for observations.

Correct!

Physical count of inventories.

Question 60
1 / 1 pts
The most critical aspect regarding separation of duties within information systems is
between

Project leaders and programmers

Correct!

Programmers and computer operators


Programmers and systems analysts

Data control and file librarians

Question 61
1 / 1 pts
Honesty, CPA is assigned to perform tests of controls over authorization of cash
disbursements. Which of the following sampling methods would be most appropriate for
Honesty to use?

Ratio Estimation

Correct!

Attributes Sampling

Difference Estimation

Variables Sampling

Question 62
1 / 1 pts
Which of the following is/are correct?
Statement I. The relationship of tolerable deviation rate to sample size is direct.
Statement II. The relationship of acceptable level of risk of assessing control risk too low
to sample size is inverse.
Correct!

II
I

None of the choices

Both

Question 63
0 / 1 pts
Hazel, CPA, vouches a sample of entries in the voucher register to the supporting
documents to assess control risk for purchases. Which assertion would this test of
controls most likely support?

Valuation or allocation

Rights and obligations

Correct Answer

Existence or occurrence

You Answered

Completeness

Question 64
1 / 1 pts
There is an inverse relationship that exist between the acceptable level of detection risk
and the
Correct!

Assurance provided by substantive tests

Preliminary judgments about materiality levels

Risk of falling to discover material misstatement

Risk of misapplying audit process

Question 65
1 / 1 pts
Attribute Sampling is

a type of sampling enables the auditor to estimate the peso amount of an account balance.

Correct!

a type of statistical sampling plan is used in tests of controls when the auditor is interested
in an attribute of a population or when the auditor is concerned with acceptance or
rejection of a hypothesis.

None of the choices

a type of statistical sampling plan is applied to a population when a goal is to reach a


conclusion about a population in terms of monetary value in an audit of financial
statements.
Question 66
0 / 1 pts
Which is a disadvantage of a business having a CIS environment?

Computers uniformly handle business transactions.

Correct Answer

Computers provides an audit trail which can be easily followed.

You Answered

Computers help lessen human error.

Computers can quickly process high volume of data.

Question 67
1 / 1 pts
Internal control is ineffective when computer department personnel

Participate in computer software acquisition decisions.

Correct!

Originate changes in master file.

Provide physical security for program files.

Design documentation for computerized systems.


Question 68
1 / 1 pts
Faith, CPA wants to determine whether she will use a statistical sampling over non-
statistical sampling. Which of the following is a correct statement that will make Faith
choose statistical sampling over non-statistical sampling?

In using the statistical sampling, Faith as will no longer need to project the sample’s
misstatements to the population.

Correct!

In using the statistical sampling, Faith will be able to quantify sampling risk.

In using the statistical sampling, Faith will be able to quantify the appropriateness and
sufficiency of audit evidence.

In using the statistical sampling, Faith will be able to test the operating effectiveness of
the client’s internal control.

Question 69
0 / 1 pts
Variables Sampling is
You Answered

All of the choices

a type of sampling enables the auditor to estimate the peso amount of an account balance.

Correct Answer

None of the choices


a type of statistical sampling plan is used in tests of controls when the auditor is interested
in an attribute of a population or when the auditor is concerned with acceptance or
rejection of a hypothesis.

Question 70
1 / 1 pts
The substantive procedures performed by the auditor relating to the financial statement
closing process shall include:
I. Agreeing or reconciling the financial statements with the underlying accounting records
II. Examining material journal entries and other adjustments made during the course of
preparing the financial statements

Correct!

I and II

II

None of the above

Question 71
0 / 1 pts
In which of the following circumstances would the use of the negative form of accounts
receivable confirmation most likely be justified?
Correct Answer

A small number of accounts may be in dispute and the accounts receivable balance arises
from sales to many customers with small balances.
A substantial number of accounts may be in dispute and the accounts receivable balance
arises from sales to a few major customers.

A small number of accounts may be in dispute and the accounts receivable balance arises
from sales to a few major customers.

You Answered

A substantial number of accounts may be in dispute and the accounts receivable balance
arises from sales to many customers with small balances.

Question 72
0 / 1 pts
Which of the following is an example of general computer control?

Reasonableness Test

Control total

Correct Answer

Disaster recovery plan

You Answered

Limit Test

Question 73
1 / 1 pts
Which of the following controls is a processing control that reconciles the input control
totals with the totals of items that have updated the file?
Validity tests

Sequence tests

File and operator controls

Correct!

Run-to-run totals

Question 74
1 / 1 pts
The understanding of the internal control relating to computer information systems
includes matters such as:
I. The significance and complexity of computer processing in each significant accounting
application. Significance relates to the materiality of the financial statement assertions
affected by computer processing.
II. The organizational structure of the client’s computer information systems activities and
the extent of concentration or distribution of computer processing throughout the entity,
particularly as they may affect segregation of duties.
III. The availability of data. Source documents, specific computer files, and other evidential
matter that may be required by the auditor may exist for only a short period or only in
machine-readable form.

I, II

Correct!

I, II, III
None of the choices

Question 75
0 / 1 pts
As one of the year-end audit procedures, the auditor instructed the client's personnel to
prepare a standard bank confirmation request for a bank account that had been closed
during the year. After the client's treasurer had signed the request, it was mailed by the
assistant treasurer. What is the major error in this audit procedure?

Sending the request was unnecessary because the account was closed before the year-
end.

Correct Answer

The confirmation request was signed by the treasurer.

You Answered

The CPA did not sign the confirmation request before it was mailed.

The request was mailed by the assistant treasurer.

Question 76
1 / 1 pts
Unauthorized alteration of on-line records can be prevented by employing:

Computer matching.

Computer sequence checks.


Correct!

Data base access controls

Key verification.

Question 77
1 / 1 pts
If the auditor decides to conduct substantive procedures at interim date, the auditor shall
cover the remaining period by performing:
I. Substantive procedures, combined with tests of controls for the intervening period
II. If the auditor determines that it is sufficient, further substantive procedures only, that
provide a reasonable basis for extending the audit conclusions from the interim date to
the period end

II

Correct!

I and II

None of the above

Question 78
1 / 1 pts
The use of computers will result in an increased number of individuals having access to the
computer. To compensate for this, a control that is used to avoid unapproved access to
sensitive programs.
Correct!
Log ins and passwords for each user

Segregation of duties within CIS department

Record count

Data Disaster Recovery

Question 79
1 / 1 pts
Monetary-unit sampling is said to eliminate the need to stratify the sample because

The risk of incorrect acceptance is inversely related to sample size

Correct!

Sample items are selected in proportion to their peso amount

The upper limit on misstatements can be computed based on statistical principles

Tolerable misstatement is considered when determining sample size

Question 80
1 / 1 pts
Which of the following statements is/are correct?
Statement I: An auditor who decided to perform test of controls would like to obtain
evidence to support his assessment of control risk at less than high level.
Statement II: The auditor may choose not to perform test of controls even if the
substantive procedures alone cannot provide sufficient appropriate audit evidence at the
assertion level.

Neither I nor II

Correct!

I only

II only

both I and II

Question 81
0 / 1 pts
Benjamin, CPA used test data approach to test the computerized system of SB Company.
Which of the following statement correct regarding test data approach?
You Answered

Test data must consist all possible valid and invalid conditions

The dummy file should be permanent in the system of the client.

Correct Answer

Test data are processed by the client’s computer under the auditor’s control.

The program tested is different from the program used throughout the year by the client.
Question 82
1 / 1 pts
It refers to plans made by the entity to obtain access to comparable hardware, software,
and data in the event of their failure, loss, or destruction.
Correct!

Back-up

Encryption

Wide Area Network (WAN)

Anti-virus

Question 83
1 / 1 pts
Which of the following statements is/are correct?
Statement I: Oral representation by the client management is not a valid evidence.
Statement II: Audit evidence obtained directly by the auditor is more reliable than that
one provided by the client management.

both I and II

Neither I nor II

I only

Correct!
II only

Question 84
1 / 1 pts
Which of the following is the best explanation of the difference, if any, between audit
objectives and audit procedures?

Audit procedures establish broad general goals, audit objectives specify the detailed work
to be performed.

Correct!

Audit objectives define specific desired accomplishments; audit procedures provide the
means of achieving audit objectives.

Audit procedures and audit objectives are essentially the same.

Audit objectives are tailor-made for each assignment, audit procedures are generic in
application.

Question 85
1 / 1 pts
Which of the following is not one of the essential concepts of internal controls?

It is effected by those charged with governance, management, and other personnel in an


entity.

It is a means or tool used by management to achieve the entity’s objectives.

Correct!
It can be expected to absolute assurance regarding that the achievement of the entity’s
objectives.

It is a process.

None of the choices.

Question 86
1 / 1 pts
If the size of the sample to be used in a particular test of attributes has not been
determined by utilizing statistical concepts but the sample has been chosen in accordance
with random selection procedures.
Correct!

The auditor may or may not achieve desired allowance for sampling risk at the desired
level of confidence

The auditor has committed a nonsampling error

No inferences can be drawn from the sample

The auditor will have to evaluation the results by reference to the principles of discovery
sampling

Question 87
1 / 1 pts
When auditing the allowance for uncollectible accounts, the least reliance should be
placed on which of the following?
An aging of past due accounts.

Ratios that show the past relationship of the allowance to net credit sales.

Correct!

The credit manager's opinion.

Collection experience of the client's collection agency.

Question 88
0 / 1 pts
When performing a substantive test of details for the existence and valuation assertions
of accounts receivable, Albert, the auditor, concluded that the sample results supported
the conclusion that the accounts receivable was materially misstated. However, it in fact
not materially misstated. This situation illustrates the risk of:

Assessing control risk too high

Correct Answer

Incorrect rejection

Assessing control risk too low

You Answered

Incorrect acceptance

Question 89
0 / 1 pts
Classical Variables Sampling is

None of the choices

You Answered

a type of statistical sampling plan is applied to a population when a goal is to reach a


conclusion about a population in terms of monetary value in an audit of financial
statements.

Correct Answer

a type of sampling enables the auditor to estimate the peso amount of an account balance.

a type of statistical sampling plan is used in tests of controls when the auditor is interested
in an attribute of a population or when the auditor is concerned with acceptance or
rejection of a hypothesis.

Question 90
0 / 1 pts
Which of the following statements is/are correct?
Statement I: Substantive procedures are required to gather evidence in respect to all
material classes of transactions, account balances, and disclosures.
Statement II: Substantive procedures are designed to detect material monetary errors or
fraud.
Correct Answer

both I and II

You Answered

I only
Neither I nor II

II only

Question 91
1 / 1 pts
Which of the following is an access control?

Back-up recovery plan

Integrated test facility

Use of power generator

Correct!

6-digit passcode

Question 92
0 / 1 pts
Generalized audit software is a computer-assisted audit technique. It is one of the widely
used technique for auditing computer application systems. Generalized audit software is
most often used to
You Answered

All of the choices.

Process data fields under the control of the operation manager.


Verify computer processing.

Correct Answer

Independently analyze data files.

Question 93
1 / 1 pts
Ivy, CPA was assigned to perform substantive test of details about the account balances
of accounts receivables. There are total of 40,000 accounts receivable and the total book
value of the accounts receivables is ₱200,000. Ivy has selected and audited a sample of
200 accounts with total book value of ₱1,040 and found that that the total audited value
of the sample is ₱840. Using ratio estimation sampling, what is the estimated total audited
value?

₱168,000

₱200,000

Correct!

₱161,538

₱160,000

Question 94
0 / 1 pts
Example of factors influencing sample size for test of details includes:
I. An increase in the auditor’s assessment of the risk of material misstatement, the larger
the sample size needs to be.
II. An increase of the auditor’s reliance on other substantive procedures directed on the
same assertion the smaller the sample size can be. Since the auditor will be using another
substantive procedure for a certain assertion of a particular account or transaction, the
auditor may choose to lower the sample size.
III. An increase of tolerable misstatement, the lower the sample size can be. A tolerable
misstatement is the application of performance materiality to a particular sampling
procedure. Accordingly, it is the maximum estimate of monetary misstatement that may
exist in an account balance or class of transactions, that even when such misstatement is
aggregated with other individual misstatements, the financial statement as whole will still
not be materially misstated.
IV. An increase in the expected amount of misstatements in the entire population the
larger the sample size needs to be so that the sample could reasonably reflect the actual
amount of misstatements in the population.
V. When a population is appropriately stratified, the aggregate of the sample sizes from
the strata will generally be lesser compare when no stratification was done to a
population.
You Answered

I, II, III, IV

II, IV, V

None of the choices

I, III, V

Correct Answer

I, II, III, IV, V

Question 95
1 / 1 pts
If the auditor is concerned that a population may contain exceptions, the determination of
a sample size sufficient to include at least one such exception is a characteristic of
Peso-unit sampling

Correct!

Discovery sampling

Variables sampling

Random sampling

Question 96
1 / 1 pts
Which of the following computer-assisted auditing techniques processes client input data
on a copy of a program that the client uses under the auditor’s control to test controls in
the computer system?

Blackbox approach

Correct!

Controlled reprocessing

Integrated test facility

Test data approach

Question 97
0 / 1 pts
If all other factors specified in a sampling plan remain constant, increasing the acceptable
risk of incorrect acceptance would cause the requires ample size to
Correct Answer

Decrease

You Answered

Increase

Become indeterminate

Remain the same

Question 98
0 / 1 pts
If no changes have occurred since the controls were last tested, a CPA should
Correct Answer

Rely on the prior year audit’s assessment of internal controls and use this assessment in
the current year.

Rely entirely on the performance of substantive audit procedures.

You Answered

Test the operating effectiveness of such controls at least once in every third audit.

Test the operating effectiveness of such controls at least once in every fourth audit.
Question 99
0 / 1 pts
Auditors who prefer statistical sampling to non-statistical sampling may do so because
statistical sampling helps the auditor

Minimize the failure to detect a material misstatement due to non-sampling risk.

Reduce the level of tolerable error to a relatively low amount

You Answered

Measure the sufficiency of the evidential matter obtained

Correct Answer

Eliminate subjectivity in the evaluation of sampling results

Question 100
1 / 1 pts
Examples of Non-Statistical Sampling includes:
I. Haphazard Selection
II. Block Selection

None of the choices

Correct!

Both

II
I

Question 101
1 / 1 pts
Types of control operating in computer information system environment includes:
I. General Controls
II. Special Controls
III. Application Control

I, III, III

I, II

II, III

Correct!

I, III

None of the choices

Question 102
0 / 1 pts
What is an auditor’s evaluation of a statistical sample for attributes when a test of 400
documents results in 16 deviations if the tolerable rate is 5%, the expected population
deviation rate is 3% and the allowance for sampling risk is 2%?
You Answered
Accept the sample results as support for planned reliance on the control because the
tolerable rate less the allowance for sampling risk equals the expected population
deviation rate.

Correct Answer

Modify planned reliance on the control because the sample deviation rate plus allowance
for sampling risk exceeds the tolerable rate.

Accept the sample results as support for planned reliance on the control because the
expected population deviation rate plus allowance for sampling risk equals the tolerable
rate.

Modify planned reliance on the control because the tolerable rate plus the allowance for
sampling risk exceeds the expected population deviation rate.

Question 103
1 / 1 pts
A negative confirmation request is where:
Correct!

the confirming party responds only to the auditor if the external party disagrees with the
information provided in the requests.

All of the choices

None of the choices

the confirming party respond directly to the auditor indicating whether the confirming
party agrees or disagrees with information contained in the requests.
Question 104
1 / 1 pts
The following are some of the types computer-assisted audit techniques:
I. Test Data Approach
II. Integrated Test Facility
III. Parallel Simulation
IV. Controlled Reprocessing
V. Embedded Audit Module

I, II, III, IV

None of the choices

I, II

Correct!

I, II, III, IV, V

Question 105
0 / 1 pts
Joy, CPA wants to evaluate whether the results of her mean-per-unit sampling will accept
or reject the clients book value amounting to ₱750,000 of the account balance she’s
examining. She obtained and set-up the following figures:

• Tolerable misstatement is ₱40,000


• Adjusted Allowance for Sampling Risk is ₱25,240
• The Estimated Total Audited Value is ₱710,000
Given the values above, evaluate whether Joy will accept the client’s book value of
₱750,000 as not materially misstated or reject it.

Joy will accept the client’s book value ₱750,000 since the misstatement of ₱40,000
(₱750,000-₱710,000) is within the tolerable misstatement of ₱40,000.

Joy will reject the client’s book value of ₱750,000 since the misstatement is above the
adjusted allowance of ₱25,240.

Correct Answer

Joy will reject the client’s book value of ₱750,000 since the total book value is not within
the acceptable interval of ₱684,750 to ₱735,240.

You Answered

Joy will accept the client’s book value of ₱750,000 since the total book value is within the
interval of the estimated total audited value and tolerable misstatement which is
₱670,000 to ₱750,000.

Question 106
1 / 1 pts
An advantage of using statistical sampling is that such techniques

Have been established in the courts to be superior to nonstatistical sampling

Correct!

Mathematically measure risk

Eliminate the need for judgmental decision

Define the value of reliability necessary to provide audit assurance


Question 107
0 / 1 pts
Which of the following techniques would an auditor use if he wants to capture the
auditee’s data transactions being processed and continuously test the entity’s
computerized information system?

Test data

You Answered

Snapshot

Correct Answer

Implanted audit module

Integrated test facility

Question 108
1 / 1 pts
Which of the following statements is/are correct?
Statement I. Even if the auditor’s assessed level of risks of material misstatement is
significantly low, the auditor shall perform substantive procedures.
Statement II. Substantive procedures comprise of Test of Details and Substantive
analytical procedures.

II only

Correct!

both I and II
I only

Neither I nor II

Question 109
1 / 1 pts
Testing the input and output of a computer system instead of the computer program itself
will

Identify all program errors, regardless of the nature of the output.

Not give confidence to the auditor in the results of the subsequent audit procedures.

Correct!

Unable to discover program errors which are not present in the output sample.

Give the auditor with the same type of evidence.

Question 110
1 / 1 pts
There are three categories of financial statement assertions: Assertions pertaining to
account balances at period end, assertions pertaining to classes of transactions and
events during the period, and assertions pertaining to presentation and disclosure. Which
of the following is a financial statement assertion that is common to all three categories?

Existence

Correct!
Completeness

Occurrence

Classifications

Question 111
0 / 1 pts
Caroline, CPA wants to test the control of the company of using passwords and user
identification for access. Which of the following procedures MOST LIKELY achieved
Caroline’s objective?
Correct Answer

Caroline should attempt to access in the computer using invalid password.

Get written representations from employees that they do not use the password for
malicious access.

Caroline should use password hacking virus to see if it can access the system.

You Answered

Review samples of transactions that were processed in the computer with a password.

Question 112
0 / 1 pts
Georgia, CPA was assigned to perform substantive test of details about the account
balances of accounts receivables. There are total of 1,000 accounts receivable and the
total book value of the accounts receivables is ₱20,025. Georgia has selected and audited
a sample of 65 accounts with total book value of ₱1,235 and found that that the total
audited value of the sample is ₱1,261. Using mean-per-unit sampling, what is the
estimated total audited value?

None of the choices

₱20,240

Correct Answer

₱19,400

₱19,760

You Answered

₱12,600

Question 113
1 / 1 pts
This method enables the estimation of the value of a population by using the average
difference between audited amounts and individual recorded.

None of the choices

Ratio Estimation

Correct!

Difference estimation

Mean-per-unit estimation
Question 114
1 / 1 pts
Which of the following statements is correct concerning statistical sampling in tests of
controls?

The expected population deviation rate has little or no effect on determining sample size
except for very small populations

As the population size doubles, the sample size also should double

Correct!

The population size has little or no effect on determining sample size except for a very
small population.

For a given tolerance rate, a larger sample size should be selected as the expected
population deviation rate decreases

Question 115
1 / 1 pts
Substantive Test Sampling Risks includes:
I. Risk of incorrect rejection (alpha risk, type I error)
II. Risk of incorrect acceptance (beta risk, type II error)

None of the choices

Correct!

Both
I

II

Question 116
1 / 1 pts
Which of the following statements is/are correct?
I. For the purpose of audit sampling in test of controls, “error” refers to deviations.
II. For the purpose of audit sampling in substantive tests, “error” refers to misstatements.
III. Based on PSA 530, if a class of transactions or account balance has been divided into
strata, the misstatement is projected collectively for all strata.

None of the choices

All of the choices

II, III

I, III

Correct!

I, II

Question 117
0 / 1 pts
In sampling plan for test of details, which of the following items have a direct relationship
with sample size?
Tolerable error

Population size

You Answered

Acceptable risk of incorrect acceptance

Correct Answer

Variation within the population

Question 118
1 / 1 pts
Substantive Procedures are audit procedures designed to detect material misstatements
at the:
Correct!

assertion level.

company level

financial statements level

None of the choices

Question 119
1 / 1 pts
The computer process whereby data processing is performed concurrently with a
particular activity and the results are available soon enough to influence the course of
action being taken or the decision being made is called:
Correct!

On-line, real-time system

Integrated data processing

Random access sampling

Batch processing system

Question 120
1 / 1 pts
Which of the following statements is/are correct?
Statement I. The auditor shall design and implement overall responses to address the
assessed risks of material misstatement at the financial statement level.
Statement II. According to PSA 330, a substantive procedure is an audit procedure
designed to detect material misstatements at the assertion level

II only

Correct!

both I and II

I only
Neither I nor II

Question 121
1 / 1 pts
Statistical sampling cannot

Select a sample draw inference about a population

Measure the risk that a sample is not representative of a population

Determine the reliability of samples

Correct!

Assure the sample will be representative of a population

Question 122
0 / 1 pts
Which of the following statements is/are correct?
As provided in PSA 530, examples of nonsampling risk include use of inappropriate audit
procedures, or misinterpretation of audit evidence and failure to recognize a
misstatement or deviation.
I. Tolerable rate of deviation is the monetary amount set by auditor in respect of which the
auditor seeks to obtain an appropriate level of assurance that such amount is not
exceeded by the actual misstatement in the population.
II. Non-sampling risk is the risk that the auditor reaches an erroneous conclusion for any
reasons which are somehow related to sampling risk.
You Answered

None of the choices


I

Correct Answer

I, II

II

Question 123
1 / 1 pts
A retailing entity uses the Internet to execute and record its purchase transactions. The
entity's auditor recognizes that the documentation of details of transactions will be
retained for only a short period of time. To compensate for this limitation, the auditor
most likely would:

Compare a sample of paid vendors' invoices to the receiving records at year-end.

Plan for a large measure of tolerable misstatement in substantive tests.

Increase the sample of transactions to be selected for cutoff tests.

Correct!

Perform tests several times during the year, rather than only at year-end.

Question 124
1 / 1 pts
To gain assurance that all inventory items in a client’s inventory listing schedule are valid,
an auditor most likely would vouch
Inventory tags noted during the auditor’s observation to items listed in receiving reports
and vendors’ invoices.

Correct!

Items listed in the inventory listing schedule to inventory tags and the auditor’s recorded
count sheets.

Items listed in receiving reports and vendors’ invoices to the inventory listing schedule.

Inventory tags noted during the auditor’s observation to items listed in the inventory
listing schedule.

Question 125
1 / 1 pts
Joy, CPA is determining the sample size for an attribute sampling. Which of the following
factors will she consider in determining the sample size for a test of control?

The risk of incorrect rejection

Correct!

Expected population deviation rate

The ratio between the sample audit values and sample book values

The audit opinion to be issued

Question 126
1 / 1 pts
A sample in which every possible combination of items in the population has an equal
chance of constituting the sample is a

Statistical sample

Correct!

Random sample

Judgment sample

Representative sample

Question 127
1 / 1 pts
Which of the following approaches of gathering evidence is considered as most effective?
Correct!

100% examination

Audit sampling

Test of reasonableness

Items for specific testing

Question 128
1 / 1 pts
Which of the following is considered a disadvantage when doing an entry in the journal
when the auditee uses a computerized system?

Transactions are authorized first before they recorded.

Random error is more common when similar transactions are processed in different ways.

Correct!

Access and alteration of files is much easier for unauthorized persons.

Trans placement errors are difficult to detect.

Question 129
1 / 1 pts
Consists of generalized computer programs designed to perform common audit tasks or
standardized data processing functions.
Correct!

Package or generalized audit software

Utility programs

System management programs

Customized or purpose-written programs

Question 130
1 / 1 pts
A positive confirmation request is where:
Correct!

the confirming party respond directly to the auditor indicating whether the confirming
party agrees or disagrees with information contained in the requests.

None of the choices

All of the choices

the confirming party responds only to the auditor if the external party disagrees with the
information provided in the requests.

Question 131
0 / 1 pts
Which of the following statements is/are correct?
Statement I: In parallel simulation method, the client’s accounting system is being used by
the auditor to reprocess the transactions.
Statement II: When using the integrated facility approach, the auditor uses a general audit
software or a customized program.
Correct Answer

Neither I nor II

II

Both

You Answered
I

Question 132
1 / 1 pts
The most common method used by auditors is the test data method, in this method the
auditor tests the:

Input data accuracy

End-product reports

Segregation of duties

Correct!

Program Processes

Question 133
1 / 1 pts
Which of the following is/are correct?
Statement I. In designing the further audit procedures to be performed, the auditor shall
obtain more persuasive audit evidence the lower the auditor’s assessment of risk.
Statement II. According to PSA 330, the auditor may decide to test the operating
effectiveness of controls at the same time as evaluating their design and determining that
they have been implemented.

I only

Neither I nor II
both I and II

Correct!

II only

Question 134
1 / 1 pts
Which of the following statements is/are correct?
Statement I: An increase in the reliance on substantive tests usually mean that the
reliance placed on internal controls decreases.
Statement II: An auditor evaluates the existing system of internal control in order to
determine the extent of substantive tests which must be performed.
Correct!

both I and II

I only

II only

Neither I nor II

Question 135
1 / 1 pts
Which of the following statements concerning sample size is true?

An increase in the tolerable occurrence rate, other factors remaining unchanged,


increases sample size.
Correct!

The higher the expected occurrence rate, other factors remaining unchanged, the larger
will be the sample size.

The more critical the attribute being tested, the higher will be the tolerable occurrence
rate set by the auditor, and the larger will be the sample size.

The lower the acceptable risk of underassessment of control risk, the smaller will be the
sample size.

Question 136
1 / 1 pts
When designing and performing substantive analytical procedures, either alone or in
combination with tests of details, as substantive procedures in accordance with PSA 330
(Redrafted), the auditor shall:
I. Determine the suitability of particular substantive analytical procedures for given
assertions, taking account of the assessed risks of material misstatement and tests of
details, if any, for these assertions.
II. Evaluate the reliability of data from which the auditor’s expectation of recorded
amounts or ratios is developed, taking account of source, comparability, and nature and
relevance of information available, and controls over preparation.
III. Develop an expectation of recorded amounts or ratios and evaluate whether the
expectation is sufficiently precise to identify a misstatement that, individually or when
aggregated with other misstatements, may cause the financial statements to be materially
misstated
IV. Determine the amount of any difference of recorded amounts from expected values
that is acceptable without further investigation.

I, II

I, II
Correct!

I, II, III, IV

I, II, III

Question 137
0 / 1 pts
Which of the following types of evidence would an auditor most likely explore to
determine whether internal control is operating as designed?

Indexes regarding the client’s industry

Budget to actual expenditure reports

Correct Answer

Client manuals showing procedures on how to use the computer programs.

You Answered

Negative confirmations of payables verifying account balances.

Question 138
1 / 1 pts
Which of the following is/are correct?
Statement I. Audit sampling is commonly used for inquiry.
Statement II. Audit sampling is commonly used for analytical procedures.

I
II

Both

Correct!

None of the choices

Question 139
1 / 1 pts
These are controls that apply to all functions of computer information system and affect
many different software applications.

Output Controls

Processing Controls

Input Controls

Correct!

General Controls

Question 140
1 / 1 pts
Which of the following is/are correct?
Statement I. Confirmation is relevant to form evidence with regard to assertion about
existence.
Statement II. The use of prenumbered invoices, then accounting for their numeric
sequence, will prove valuation assertion.
II only

Neither I nor II

Correct!

I only

I and II

Question 141
0 / 1 pts
Philippine Standards on Auditing requires the auditor to design and perform a substantive
procedure for:
each class of transaction, account balance, and disclosure.
each material class of transaction, account balance, and disclosure.
each trivial class of transaction, account balance, and disclosure.
Both B and C
You Answered

each class of transaction, account balance, and disclosure

each trivial class of transaction, account balance, and disclosure.

Both B and C

Correct Answer

each material class of transaction, account balance, and disclosure.


Question 142
1 / 1 pts
An auditor is evaluating the results of a variables sampling plan. Which of the following is
not relevant to the auditor’s judgment about the sample?
Correct!

Management’s explanations for why errors in the sample occurred

Qualitative information that lends insight into errors fraud

Considering the effects of sampling risk

Projecting the sample error to the population

Question 143
1 / 1 pts
If an auditor concludes that internal controls are likely to be effective, the preliminary
assessment of control risk can be reduced, leading to a(n)_____ the acceptable risk of
incorrect acceptance

reduction in

Correct!

increase in

increase of decrease
elimination of

Question 144
0 / 1 pts
External confirmations can be in:

You Answered

negative request form

None of the choices

Correct Answer

positive and negative request form

positive request form

Question 145
0 / 1 pts
In which of the following situations would the auditor not perform 100% testing?

The population constitutes a small number of large volume items

When both inherent and control risks are high and other means do not provide sufficient
appropriate audit evidence.

Correct Answer

The population constitutes a large number of small volume items.


You Answered

The repetitive nature of a calculation or other process performed automatically by an


information system makes a 100% examination cost effective.

Question 146
0 / 1 pts
Tracing is:

None of the choices

Correct Answer

an audit procedure that involves tracing an accounting information from source document
to another subsequent documents like journal or general ledger.

You Answered

an audit procedure that involves checking the mathematical accuracy of document and
records.

an audit procedure establishing agreement between separate sources of information such


accounting records with bank statements.

Question 147
1 / 1 pts
ABCD Company has changed from a manual system to a computerized payroll system in
which employees record time in and out with magnetic proximity cards. The computer
system automatically updates all payroll records. Because of this change

Employee collusions are eliminated.

Integrated Test Facility must be used.


The potential for payroll-related fraud is diminished.

Correct!

Part of the audit trail is altered.

Question 148
0 / 1 pts
Analytical procedures also include consideration of relationships, for example:
I. Among elements of financial information that would be expected to conform to a
predictable pattern based on the entity’s experience.
II. Between financial information and relevant non-financial information.

None of the choices

You Answered

II

Correct Answer

I, II

Question 149
1 / 1 pts
Halsey, CPA was assigned to perform substantive test of details about the account
balances of accounts receivables. There are total of 1,000 accounts receivable and the
total book value of the accounts receivables is ₱20,000. Halsey has selected and audited a
sample of 50 accounts with total book value of ₱950 and found that that the total audited
value of the sample is ₱980. Using difference estimation sampling, what is the estimated
total audited value?
Correct!

₱19,600

₱19,400

None of the choices

₱20,600

₱20,400

Question 150
1 / 1 pts
To obtain evidence that online access controls are properly functioning, an auditor most
likely would

Vouch a random sample of processed transactions to assure proper authorization

Examine the transaction log to discover whether any transactions were lost or entered
twice due to a system malfunction

Correct!

Enter invalid identification numbers or passwords to ascertain whether the system rejects
them.
Create checkpoints at periodic intervals after live data processing to test for unauthorized
use of the system.

Quiz Score: 107 out of 150

You might also like